You are on page 1of 64

Advanced Accounting

CAP-II, June 2011

Maximum Marks - 100

Total No. of Questions - 6

Time Allowed - 3 Hours


Marks

Attempt all questions. Working notes should form part of the answer.

1. Sudeep and Mayur had been carrying business independently. They agree to
amalgamate and form a new company Sumeru Ltd. with an authorized share
capital of Rs. 2,00,000 divided in to 40,000 equity shares of Rs. 5 each..
On 31st Ashad 2066, the respective balance Sheet of Sudeep and Mayur were as
follows:

Description Sudeep Rs Mayur Rs.


Fixed Assets 3,17,500 1,82,500
Current assets 1,63,500 83,875
4,81,000 2,66,375
Less: Current Liabilities 2,98,500 90,125
Representing Capital 1,82,500 1,76,250

Additional information:
a) Revalued figures of Fixed assets were Sudeep Rs 3,55,000 and Mayur Rs.
1,95,000 and revalued figure of current Assets were, Sudeep Rs. 1,49,750 and
Mayur Rs. 78,875.
b) The debtors of Sudeep includes Rs. 21,675 to be received from Mayur.
c) The purchase consideration is satisfied by issue of the following shares and
debentures:
d) 30,000 equity shares of Sumeru Ltd to Sudeep and Mayur in the proportion of
the profitability of their respective business based on the average net profit
during the last three years which were as follows:

Sudeep Rs Mayur Rs
2064 Profit 2,24,788 1,36,950
2065 Loss / Profit ( 1,250) 1,71,050
2066 Profit 1,88,962 1,79,500

e) 15% Debenture in Sumeru Ltd. at par to provide an income equivalent to 8 %


return on capital employed in their respective business as on 31st Ashad 2066
after revaluation of assets.

You are required to:


i) Compute the amount of Debentures and shares to be issued to Sudeep and
Mayur
ii) A Balance Sheet of Sumeru Ltd. showing the position immediately after
amalgamation.

Answer No.1

NMG P.T.O.
(2)

Sudeep Rs Mayur Rs
(1) Average Profit:
Rs (2,24,788-1,250+1,88,962)/3 1,37,500
Rs ( 1,36,950+1,71,050+1,79,500)/3 1,62,500
(2) Equity Shares issued:
(a) Ratio of distribution 1375 1625
(b) No. of shares 13,750 16,250
(c) Amount:
13,750@ Rs. 5 68,750
16,250 @ Rs. 5 81,250

(3) Capital employed (after revaluation of assets)


Fixed assets 3,55,000 1,95,000
Current Assets 1,49,750 78,875
5,04,750 2,73,875
Less: Current Liabilities 2,98,500 90,125
2,06,250 1,83,750

(4)Debenture Issued
8% debenture on capital employed 16,500 14,700
15% Debenture to be issued to provide equivalent income
Sudeep Rs 16,500*100/15 1,10,000
Mayur Rs 14,700*100/15 98,000

Working No.1 Calculation of Purchase Consideration:


Description Sudeep Rs Mayor Rs Total Rs.

Equity shares issued 68,750 81,250 1,50,000


15% Debenture issued 1,10,000 98,000 2,08,000
1,78,750 1,79,250 3,58,000

WN 2: Calculation of Capital Reserve

(a) Net Assets taken over:


Fixed Assets 3,55,000 1,95,000 5,50,000
Current assets 1,28,075 78,875* 2,06,950
4,83,075 2,73,875 7,56,950
Less: Current Liabilities 2,98,500 68,450** 3,66,950
184,575 2,05,425 3,90,000

(b) Purchase Consideration 1,78,750 1,79,250 3,58,000

(c ) Capital Reserve (a-b) 5,825 26,175 32,000

* Rs 1,49,750- Rs 21,675 ** Rs 90,125- Rs 21,675


(3)

Balance Sheet of Sudeep Ltd as on 31st Ashad 2066

Rs.
Fixed Assets 5,50,000
Current assets, Loans & Advances Rs. 2,06,950
Less; Current Liabilities Rs 3,66,950 ( 1,60,000)
3,90,000

Represented by:
Share Capital** 1,50,000
Reserve & Surplus 32,000
15% Debenture 2,08,000 3,90,000

**
Share Capital:
Authorised share capital 40,000 shares of Rs 5 2,00,000
Issued and subscribed capital:
30,000 equity shares of Rs 5 each 1,50,000
( all the above shares have been allotted as fully
Paid up without payments being received in cash.)

2.
a) Media Pvt. Ltd. obtained advertisement rights for One Day World Cup
Cricket Tournament to be held in March/April, 2011 for Rs. 50,00,000.
By 31st January, 2011 they have paid Rs. 30,00,000 to secure these
advertisement rights. The balance Rs. 20,00,000 was paid in February, 2011.
By 31st January, 2011 they procured advertisement for 70% of the available
time for Rs. 70,00,000. The advertisers paid 60% of the amount by that date.
The balance 40% was received in February, 2011.
Advertisements for the balance 30% time were procured in February, 2011 for
Rs. 30,00,000. The advertisers paid the full amount while booking the
advertisement.
75% of the advertisement time is expected to be available in March, 2011 and
the balance 25% in April, 2011.
You are asked to:
i) Pass journal entries in relation to the above.
ii) Show in columnar from as to how the items will appear in the monthly
financial statements for January, February, March and April 2011. Give
reasons for your treatment. (6+4=10)

b) Ramesh and Suraj decided to work in partnership in the following scheme,


agreeing to share profit as under:
Ranesh 3/4th share
Suraj 1/4th share

They guaranteed the subscription at par of 10,00,000 shares of Rs.10 each in


Sheetal ltd and to pay all expenses up to allotment in consideration of Sheetal
(4)

Ltd. furthermore 50,000 Share of Rs. 10 each fully paid (other than 10,00,000
Share issued to public) was issued to them. Ramesh introduced cash in to the
business to meet the following expenses:

Registration fees Rs. 48,500


Advertising charges Rs 28,500
Printing charges of Memorandum of Association and
Articles of Association and Prospectus etc. Rs. 23,000

Suraj introduced cash to meet following expenses:


Rent Rs. 37,000
Legal fees Rs. 13,000

Application fell short of the 10,00,000 shares by 30,000 shares. Suraj


introduced further cash on joint account for the said 30,000 shares. This
amount was utilized to subscribe the said 30,000 shares and paid to the
company.

The guarantee having been fulfilled, Sheetal Ltd. Handed over the Ramesh
and Suraj 50,000 shares.
The partnership sold all shares. Suraj received the sale proceed of 20,000
shares amounting to Rs. 1,80,000 and Ramesh of the remaining 60,000 shares
amounting Rs. 5,00,000.

Give necessary accounts in the books of both parties. 10

Answer No.2
a)
In the books of Media Pvt. Ltd.
Journal Entries
Dr. Cr.
Rs. in lakhs Rs. in lakhs
2011
Jan Advance for advertisement rights (purchase) A/c Dr. 30.00
To Bank A/c 30.00
(Being advance paid for obtaining advertisement rights)
Bank A/c Dr. 42.00
To Advance for advertisement time (sale) A/c 42.00
(Being advance received from advertisers
amounting to 60% of Rs. 70 lakhs for booking
70% advertisement time)
Feb Advance for advertisement rights (purchase) A/c Dr. 20.00
To Bank A/c 20.00
(Being balance advance i.e., Rs. 50 lakhs less
Rs. 30 lakhs paid)
Bank A/c Dr. 28.00
(5)

To Advance for advertisement time (sale) A/c 28.00


(Being balance advance i.e., Rs. 70 lakhs less
Rs. 42 lakhs received from advertisers)
Bank A/c Dr. 30.00
To Advance for advertisement time (sale) A/c 30.00
(Being advance received from advertisers
in respect of booking of balance 30% time)
March Advertisement rights (purchase) A/c Dr. 37.50
To Advance for advertisement rights (purchase) A/c 37.50
(Being cost of advertisement rights used in March
i.e., 75% of Rs. 50 lakhs, adjusted against advance
paid)
Advance for advertisement time (sale) A/c Dr. 75.00
To Advertisement time (sale) A/c 75.00
(Being sale price of advertisement time in March i.e.,
75% of Rs. 100 lakhs adjusted, against advance
received from advertisers)
Profit and Loss A/c Dr. 37.50
To Advertisement rights (purchase) A/c 37.50
(Being cost of advertisement rights debited to Profit
and Loss Account in March)
Advertisement time (sale) A/c Dr. 75.00
To Profit and Loss A/c 75.00
(Being revenue recognized in Profit and Loss
Account in March)
April Advertisement rights (purchase) A/c Dr. 12.50
To Advance for advertisement rights (purchase) A/c 12.50
(Being cost of advertisement rights used in April, i.e.,
25% of Rs. 50 lakhs, adjusted against
advance paid)
Advance for advertisement time (sale) A/c Dr. 25.00
To Advertisement time (sale) A/c 25.00
(Being sale price of advertisement time availed in
April i.e., 25% of Rs. 100 lakhs, adjusted against
advance received from advertisers)
Profit and Loss A/c Dr. 12.50
To Advertisement rights (purchase) A/c 12.50
(6)

(Being cost of advertisement rights used in Apri,


debited to Profit and Loss Account in April)
Advertisement time (sale) A/c Dr. 25.00
To Profit and Loss Account 25.00
(Being revenue recognized in April)

(ii) Monthly financial statements


(1) Revenue statement (Rs. in lakhs)
Jan Feb March April
Rs. Rs. Rs. Rs.
Sale of advertisement time – – 75.00 25.00
Less: Purchase of advertisement rights – – 37.50 12.50
Net profit – – 37.50 12.50
(2) Balance sheet as at 31.1.2011 28.2.2011 31.3.2011 30.4.2011
Sources of funds:
Net profit – – 37.50 50.00
Application of funds:
Current assets, loans and advances:
Advance for advertisement rights 30.00 50.00 12.50 –
Bank Balance 12.00 50.00 50.00 50.00
42.00 100.00 62.50 50.00
Less: Current liabilities
Advance for advertisement time
(received from advertisers) 42.00 100.00 25.00 –
Net current assets – – 37.50 50.00
As per NAS 7 on Revenue Recognition, under proportionate completion method, revenue from service
transactions is recognized proportionately by reference to the performance of each act where
performance consists of the execution of more than one act.
Therefore, income from advertisement is recognized in March, 2011 (75%) and April, 2011 (25%) in
the proportion of availability of the advertisement time. (6+4=10 marks)

b)
Memorandum Joint Venture Account

To Ramesh: Rs. Rs. By Ramesh; Rs


Registration charges 48,500 Sale proceeds of
Advertising 28,500 60,000 shares 5,00,000
Printing 23,000 1,00,000
To Suraj: By Suraj:
Rent 37,000 Sale proceeds of
Legal fees 13,000 50,000 20,000 shares 1,80,000
To Suraj: subscription of 3,00,000
Unapplied for shares
To Ramesh (3/4th profit) 1,72,500
th
To Suraj ( 1/4 profit) 57,500
Total 6,80,000 Total 6,80,000
Student should be careful to make no entry for the shares received from the company as consideration
for the guarantee.

In the book of Ramesh, the account with Suraj will be as follows:


(7)

Suraj in Joint Venture Account


To Bank: Rs. By Bank: Rs.
Registration charges 48,500 Sale proceed of 60,000
Advertising 28,500 shares 5,00,000
Printing 23,000
To Profit & Loss Account
2/3 ths share of profit 1,72,500
To Bank 2,27,500
Total 5,00,000 Total 5,00,000

In the book of Suraj, the account with Ramesh will be as follows:


Ramesh in Joint VentureAccount
To Bank: Rs. By Bank: Rs.
Rent 37,000 Sale proceed of 20,000
Legal 13,000 shares 1,80,000

To Bank: Subscription of By Bank: Settlement 2,27,500


Unapplied for shares 3,00,000
To Profit & Loss Account:
1/4th share of profit 57,500
Total 4,07,500 Total 4,07,500

3.
a)
i) Find out the income to be recognized in the case of Good Faith Bank
Limited for the year ended 31st Asadh, 2067 as per the directives issued by
NRB.

(Rs. in Lakhs)

Performing Assets Non-performing Assets


Interest Realized in Interest Realized in
accrued cash accrued cash
Term Loans 540 400 450 20
Consumer Loan 250 205 360 16
2
ii) The following is an extract from the Trial Balance of Golden Bank Ltd. as
at 31st March, 2011:
Rebate on bills discounted as on 1-4-2010 341,295 (Cr.)
Discount received 850,780 (Cr.)
Analysis of the bills discounted reveals as follows:
Amount (Rs.) Due date
14,00,000 June 1,2011
43,60,000 June 8,2011
(8)

28,20,000 June 21,2011


40,60,000 July 1,2011
30,00,000 July 5,2011
You are required to find out the amount of discount to be credited to Profit
and Loss account for the year ending 31st March, 2011 and pass Journal
Entries. The rate of discount may be taken at 10% per annum. (4+4=8)

b)
i) What is entity concept? 2
ii) Following are data of Robinson
 Market Value of Stock as on Shrawan 01, 2066: Rs. 300,000
(Market Price being cost plus 20% thereof).
 Purchases for the year: Rs. 1,450,000
(Out of which goods worth Rs.150,000 was delivered on Shrawan 08,
2067)
 Sales for the year: Rs. 1,480,000.
 Goods costing Rs. 20,000 was taken by the proprietor for personal use.
 Goods costing Rs. 10,000 was used for firm‟s office stationery.
 Normally, the firm sells goods on cost plus 40%

Determine the value of stock to be taken for Balance Sheet purpose of


Robinson as on Ashadh end, 2067 8

Answer No.3
a) (i)Calculation of interest income of Good Faith Bank Ltd. to be recognized for the year ended
31st Asadh, 2067.
Term Loans Rs. in Lakhs
Interest realized in cash regarding performing assets 400
Interest realized in cash regarding non-performing assets 20 420
Consumer Loan
Interest realized in cash regarding performing assets 205
Interest realized in cash regarding non-performing assets 16 221
Total Interest to be recognized 641
(2 Marks)
(ii)
The amount of rebate on bills discounted as on 31st March, 2011 the period which has not
been expired upto that day will be calculated as follows:
Discount on Rs.14,00,000 for 62 days @ 10% 23780
Discount on Rs.43,60,000 for 69 days @ 10% 82,420
Discount on Rs.28,20,000 for 82 days @ 10% 63,355
Discount on Rs.40,60,000 for 92 days @ 10% 102,335
Discount on Rs.30,00,000 for 96 days @ 10% 78,905
Total 350,795
The amount of discount to be credited to the profit and loss account will be:
(9)

Rs.
Transfer from rebate on bills discounted as on 31.03.2010 341,295
Add: Discount received during the year 850,780
1,192,075
Less: Rebate on bills discounted as on 31.03.2011 (as above) 350,795
841,280
Journal Entries
Rs. Rs.
Rebate on bills discounted A/c Dr. 341,295
To Discount on bills A/c 341,295
(Transfer of unexpired discount on 31.03.2010)
Discount on bills A/c Dr. 350,795
To Rebate on bills discounted 350,795
(Unexpired discount on 31.03.2011 taken into account)
Discount on Bills A/c Dr. 841,280
To P & L A/c 841,280
(Discount earned in the year, transferred to P&L A/c)
i. Business consists of person and resources. Person representing the business is separate and
distinct from the business enterprises. Accounting system deals with the economic activities
of the business not of owner. In Entity Concept, preparation of Balance Sheet of the business
does not consider the personal assets and liability of the owner of the business.

ii.
Robinson
Statement showing Value of Stock
as on Ashadh end, 2067
Opening Stock ( as on Shrawan 01, 2066) 250,000
Add:
Purchases during the Year 1,450,000
Less: Goods in Transit (150,000)
Less: Goods taken by proprietor for personal use (20,000)
Less: Goods used for office's stationery (10,000) 1,270,000
Less:
Cost of Goods Sold:
Sales for the Year 1,480,000
Less: Profit on Sales (422,857)
1,057,143
Stock in Hand (as on Ashadh end, 2067) 462,857
Add:
Goods in Transit 150,000 150,000
Value of Stock to be shown in Balance Sheet (as on
Ashadh end, 2067) 612,857

Working Notes:
(10)

1. Calculation of Opening Stock:


Market Value of Stock: 300,000
Profit Margin (on Cost) 20%
Cost Price of Opening Stock: =300,000 = 250,000
(100+20)
2. Calculation of Profit on
Sales:
Sales 1,480,000
Profit Margin (on Cost) 40%
Profit Amount =1,480,000 X 40 = 422,857 1,057,143
(100+40)

4.
a) On Baishakh 01 2063, Rama & Rama took delivery of a car from Sita & Sita
on hire purchase system, Rs. 105,000 being paid on delivery and the balance
in six annual installments of Rs. 60,000 each payable at the end of each year.
The Cash Price of the car is Rs. 3,60,000. Calculate the amount of interest for
each year. 5

b) Following is the information of two business organizations belonging to same


industry for 2065/066;
Particulars Birat Industry Kirat industry
Rs Rs

Opening balance 25,000 15,000


Cash flow from operating activities 8,500 24,000
Cash from investing activities;
Sale of fixed assets 35,000 6,000
Cash from financing activities:
Amount borrowed 40,000 5,000
Closing Balance 1,08,500 50,000
Birat claims that it‟s cash generating activity is better than that of Kirat. Do
you agree ? Comment on the basis of cash generating ability of both
organization 5

c) Palpa Power company decided to replace some parts of its plant by an


improved plant. The plant to be replaced was built in 2065 for Rs.70,00,000. It
is estimated that it would cost Rs.130,00,000 to build a new plant of the same
size and capacity. The cost of the new plant as per the improved design was
Rs.2,10,00,000 and in addition, material belonging to the old plant valued at
Rs.7,60,000 was used in the construction of the new plant. The balance of the
plant was sold for Rs.6, 00,000.Compute the amount to be written off to
revenue and the amount to be capitalized. Also prepare Plant account and
Replacement account. 5
Answer No.4
a) Amount Weight
2063 Amount outstanding for interest after Down Payment 360,000 6/21
2064 Amount outstanding for interest after First Installment 300,000 5/21
(11)

Amount outstanding for interest after Second


2065 Installment 240,000 4/21
2066 Amount outstanding for interest after Third Installment 180,000 3/21
Amount outstanding for interest after Fourth
2067 Installment 120,000 2/21
2068 Amount outstanding for interest after Fifth Installment 60,000 1/21

Total Interest for all the six years is Rs. 105,000 (360000+105000-105000) which should be
divided in the ratio of 6:5:4:3:2:1 for all these years.
Hence, Interest for each year:
2063 : 105,000 X 6/21 = 30,000
2064 : 105,000 X 5/21 = 25,000
2065 : 105,000 X 4/21 = 20,000
2066 : 105,000 X 3/21 = 15,000
2067 : 105,000 X 2/21 = 10,000
2068: 105,000 X 1/21 = 5,000

b) Kirat organization has a better capacity to generate cash. Claim of Birat organization is not
appropriate. The evaluation is based on the quantum of cash flow from operating activities
rather than on investing or financing activities. Birat organization is generating funds from
either sale of goods or out of borrowing and quantum of funds so generated is not an
appropriate indicator for evaluation.

C)

(a) (i) Calculation of amount chargeable to revenue Rs. Rs.


Estimated current cost of replacing old plant 130,00,000
Less: Realization from sale of remaining of the old plant 6,00,000
Value of materials belonging to the old Plant
used in the construction of new plant 7,60,000 13,60,000
Total 116,40,000
(ii) Calculation of amount to be capitalized
Cost of building new plant (cash) 2,10,00,000
Add: Value of materials belonging to the old
plant used in the construction of the new plant 7,60,000 217,60,000
Less: Estimated current cost of replacing old plant 130,00,000
Total 87,60,000
(iii) Plant Account
Rs. Rs.
To Balance b/d 70,00,000 By Balanced c/d 157,60,000
To Cost of construction:
Cash
(210,00,000-130,00,000) 80,00,000
Cost of old material used 760,000 _________
157,60,000 157,60,000
(iv) Replacement Account
Rs. Rs.
To Bank account (portion to By Bank account 6,00,000
be written off out of the By Plant account 760,000
replacement cost) 130,00,000 By Revenue account 116,40,000
(12)

130,00,000 130,00,000

5.
a) The company deals in three products, A, B & C, which are neither similar nor
interchangeable. At the time of closing of its account for the year 2066/067,
the Historical and Net Realizable Value of the items of closing stock are
determined as follows:

Items Historical Cost Net Realizable Value


( Rs. in Lakhs) ( Rs. in Lakhs)
A 40 28
B 32 32
C 16 24

Compute the value of closing stock? 5


b) What are the conditions that have to be satisfied for recognition of revenue
from sale of goods? 5
c) A Ltd. received a grant of Rs 2 crores from the Government for purchase of a
special purpose machinery during 2062-063. The cost of machinery was Rs 38
crores and had a useful life of 9 years. During the current year 2065-066 the
grant has become refundable due to non fulfillment of certain conditions
attached to it. Assuming that the entire amount of grant was deducted from the
cost of machinery in the year of acquisition, state with reasons, the accounting
treatment to be followed in the rear 2065-066 5

Answer No.5
a) As per para 9 of NAS 4 on Inventories, inventory should be valued at the lower of cost and net
realizable value. Inventories should be written down to net realizable value on an item-by-item
basis in the given case:
Historical Net Realizable Valuation of Closing
Items Cost Value Stock
(Rs. in Lakhs) (Rs. in Lakhs) (Rs. in Lakhs)
A 40.00 28.00 28.00
B 32.00 32.00 32.00
C 16.00 24.00 16.00
88.00 84.00 76.00

Hence, closing stock will be valued at Rs. 76 lakhs

b) As per NAS 7, Revenue from Sale of goods shall be recognized when all the following
conditions have been satisfied:
i. The entity has transferred to the buyer the significant risks and rewards of ownership
of goods;
ii. The entity retains neither continuing managerial involvement to the degree usually
associated with ownership nor effective control over the goods sold;
iii. The amount of revenue can be measured reliably;
iv. It is probable that the economic benefits associated with the transaction will flow to
the entity; and
(13)

v. The cost incurred or to be incurred in respect of the transaction can be measured


reliably.

c) Value of asset (Without grant) = Rs.38 crore


Depreciation (Without Grant) = Rs.38 crore/9years
= 4.22 crore per year
As per NAS, the additional depreciation needs to be charge of expenses. Hence,
Already charge Depreciation
Book value (before refund of grant) = Rs 20 crore
(assuming SLM method and no residual value, annual depreciation will be Rs.(38-2)/9 = Rs. 4 crore,
depreciation for 4 years will be Rs.16 crore
Additional Depreciation for 4 years = (4.22×4)-(4×4)
(change as expenses for current year = 16.88-16=0.88 crore
Revises book Value = 38-16.88 =21.12 crore, Depreciation for next 5 year =4.22

6. Write short notes on:

a) Share and Stock. 4


b) Escalation Clause in Contracts 3
c) Accounting estimate. 3
Answer No.6
a) Share and Stock
In today‟s financial markets, the distinction between stocks and shares has been
somewhat blurred. Generally, these words are used interchangeably to refer to the pieces
of paper that denote ownership in a particular company, called stock certificates.
However, the difference between the two words comes from the context in which they
are used. For example, “stock” is a general term used to describe the ownership
certificates of any company, in general, and “shares” refers” to a the ownership
certificates of a particular company. So, if investors say they own stocks, they are
generally referring to their overall ownership in one or more companies. Technically, if
someone says that own shares- the question then becomes – shares in what company?
Bottom line, stocks and shares are the same thing. The minor distinction between stocks
and shares is usually overlooked, and it has more to do with syntax than financial or legal
accuracy.

b) Escalation Clause in Contracts:


This is a clause provided in the contracts to cover up any changes in the price of the
contract due to changes in the price of raw materials and labour or change in utilization of
factor of production. The object of this clause is to safeguard the interest on both sides
against unfavourable change in prices. The basis, on which the factor prices are based, is
laid down in the contract. For e.g.: in a contract with transport undertaking, it may be
provided that the price per ton km will increase or decrease for each rise or fall in price of
diesel by 10% of the prevailing prices.

c) Accounting estimate.
As a result of the uncertainties in business activities, many financial statement items
cannot be measured with precision but can only be estimated. These are called accounting
(14)

estimates. Therefore, the management makes various estimates and assumptions of


assets, liabilities, incomes and expenses as on the date of preparation of financial
statements. This process of estimation involves judgements based on the latest
information available.
Examples of estimation in some fields are:
(i) Estimation of useful life of depreciable assets.
(ii) Estimation of provision to be made for bad and doubtful debts.
The Institute of Chartered Accountants of Nepal
Suggested Answers of Auditing and Assurance
CAP II Examination – June 2011

1. Give your opinions with reasons on the following cases: (45=20)

a) Terai Bank Limited provided a long term loan amounting to Rs. 250 Million to Relax Hotel Pvt. Limited for hotel
operation. The hotel is unable to repay installment (principal & interest) as per repayment schedule. Accordingly you are
appointed by the Bank for special review of the income of the hotel to ensure whether income procedures are duly complied
with and the amount has been fully accounted for. As a special reviewer how would you plan your review work?

b) A Donor has appointed you as an auditor for ensuring financial management capacity of the NGO before funding
made to the NGO to rely on the fund operating capability of the NGO. What would be your special focus area for such
assignment?

c) The Board of Directors meeting of M/s Dikpal Commercial Bank Limited, proposed dividend @ 50 % for the fiscal
year 2066/067. The Annual General Meeting approved the dividend @ 70% instead of the proposed dividend by the Board
of Directors. Examine the decision of the Annual General Meeting to approve dividend in excess of the proposed dividend
by the Board of Directors and whether it is commensurate with the company Act, 2063.

d) Paudel Provident Fund (PPF) is a approved retirement fund, which manages retirement benefits of employees of
group of companies relating to "Poudel Group". PPF has entered into an agreement with a commercial bank for maintenance
and operation of the retirements accounts relating to its beneficiaries. Under the agreement, PPF is required to open a
deposit account in the bank and maintain the entire fund only with the bank. During the audit of PPF, auditor finds that bank
has credited its account with excess interest by 4,32,000. All the interest income in the account is distributed on pro-rata
basis to the individual employees through accounts maintained in the bank itself.

Answer No.1
a) An efficient and effective review can only be performed if this has been thoroughly and properly planned. The planning
stage of the review should be used to establish an overall strategy for the review.
Adequate planning will ensure that appropriate attention is given to crucial areas of the review and that potential problems
are identified on a timely basis. At the planning stage the engagement partner should assign the necessary staffs who
possess the skills and ability required in order to ensure the review is carried out efficiently and in accordance with the
Nepal Standards on Auditing.
Planning Activities
At the planning stage the overall audit strategy is developed. The audit strategy sets the scope, timing and direction of the
review. At this stage the reviewer will develop the detailed review plan which will help identify problem areas and
important review areas.
Once the strategy has been established, then the reviewer is able to develop the more detailed review plan to address the
matters identified in the overall strategy.
Contents of the Plan
The review plan should document the nature, timing and extent of the procedures to be adopted which should be sufficient
and effective enough to be able to assess the risks associated with review engagements.
The detailed plan should also contain a description of the nature, timing and extent of planned further audit procedures at the
assertion level for each material class of transactions, account balances and disclosures.
Finally, the review plan should also contain details of other procedures to be adopted so that the review can be carried out
effectively.
Typical contents of the detailed plan are:
• Nature of the business and what it does
• Risk and problematic areas (both business risk and financial statement risk)
• Details of any complexities associated with the assignment
• Specific accounting and auditing standards relevant to the assignment
• Budgets
• Planned review procedures
• Details of sampling techniques
• Key personnel employed at the clients
For the purpose of this review of the income of the hotel and purpose of assignment an audit program with detailed
procedures should also be developed to conduct the review work efficiently and effectively. Audit procedures inter alia
includes amongst the following:
i. Whether rack rates has been maintained for different types of rooms (Suit/Deluxe/Superior etc), banquette rates are
formalized with recipe and other rates are formalized & approved by competent authority.
ii. Whether there are formal discount rates. If discount rate is in excess of formal rate, whether this is approved by the
competent authority.

NMG P.T.O.
(16)

iii. Whether rate agreements has been entered with regular official & individual parties and duly signed of both parties. If
yes agreed rates have been applied or otherwise to ensure how the rate is formalized in such cases.
iv. Whether complimentary service provided to guests are within the power of approving authority.
v. Whether the billing are made on timely basis and accounted accordingly on departure of guest with evidence of
acceptance by guests. If bill is to be forwarded to customer through mail, is it forwarded on timely basis?
vi. Whether there is clear cut credit policy and credit facility provided only to eligible guest backing with deposition of
advance.
vii. Whether revenue as per income of auditor`s report, accounting system and VAT return has been matched. In case of
difference, is proper reconciliation has been made.
viii. Whether rates have been reviewed as per formal policy of hotel and approved by the competent authority.
ix. Whether record of revenue charged has been reflected in the concerned ledger folios of the guest.
x. Whether accounting system (Software/Manual) for revenue is commensurable with volume of transactions of the
hotel.
xi. Check the closing procedures of the hotel during physical presence on sample night audits.
xii. Make the surprise check on different point of sales and observed if any evidences of concealing the income (by not
issuing bill, by providing huge discounts, by undervaluing the bill etc.)
xiii. Check the recovery rate from debtors and utilization of cash.

b) For ensuring financial management capacity of the NGO my special focus area would be:
i. To ensure whether adequate internal control system for accounting administration has been existed within the
organization.
ii. To ensure the required experiences & academic qualification of accounting staff.
iii. To ensure the volume of donors fund previously operated by the NGO.
iv. To ensure the adequacy of policies etc (like accounting manual, chart of account, financial & administrative
regulation, software are in place & adequate).
v. To see the annual audit report, internal audit report & other types of audit report for ensuring serious and risky areas
if any to pay due attention.
vi. To ensure the basic accounting concept of non-accounting staff for handling the advances.
vii. To see overall account of the NGO, whether there is any diversion of project fund to institutional fund through any
unfair means.
viii. To see the composition of program cost & office running cost for ensuring whether office running cost mainly salary
is excessively high than program cost.
ix. To the composition of common cost (cost shared by different donors) in past period to ensure that whether same cost
has been borne by different donors in books of accounts.
x. To ensure whether periodical reconciliation (bank/cash/fund etc) system are in place.
xi. To ensure whether budgetary control system are in place.
xii. To ensure whether there is excessive turnover of the accounting staff.
xiii. To ensure whether proper & adequate delegation of authority has been made and properly exercised in practice.

c) The Annual General Meeting is authorized body to approve dividend under section 77 (6) of the Company Act,
2063. The proviso to the said section has however, restricted the Annual General Meeting to approve the dividend in
excess of the dividend proposed by the Board of Directors. However, the dividend at lower rate can be approved. Thus,
the decision of Annual General Meeting of Ms. Shangrila Commercial Bank Ltd. to approve dividend in excess of the
dividend proposed by the Board of Directors is void and Annual General Meeting can only approve the dividend at the
lower rate or at the rate proposed by the Board of Directors.

d) As per NSA 200 "Objective and General Principles Governing an Audit of Financial Statements", The objective
of an audit of financial statements is to enable the auditor to express an opinion whether the financial statements are
prepared, in all material respects, in accordance with an identified financial reporting framework.
Once the financial statements are misstated, auditor is required to report on the matter by quantifying its impact on the
financial statements. Therefore, auditor should look in to the matter by verifying the fact about how PPF has treated the
excess credit by the bank in its account. Even if bank cannot recover from PPF's employee, and the PPF has not given any
effect to this item in its account, auditor should ask management to disclose the fact in notes to the account of financial
statements and of require, should modify the audit report accordingly.

2. Answer the following:


a) What are the auditor‟s responsibilities for detection of frauds and errors? 10
b) Briefly explain the concept of Materiality. 5

Answer No. 2
a) It has been explicitly mentioned in the NSA 240 that the primary responsibility for the prevention and detection
of fraud rests with both those charged with governance of the entity and with management. The respective responsibilities
of those charged with governance and of management may vary by entity to entity. It is important that management, with
(17)

the oversight of those charged with governance, place a strong emphasis on fraud prevention, which may reduce
opportunities for fraud to take place, and fraud deterrence, which could persuade individuals not to commit fraud because
of the likelihood of detection and punishment. It is the responsibility of those charged with governance of the entity to
ensure, through oversight of management, that the entity establishes and maintains internal control to provide reasonable
assurance with regard to reliability of financial reporting, effectiveness and efficiency of operations and compliance with
applicable laws and regulations. This responsibility includes establishing and maintaining controls pertaining to the
entity‟s objective of preparing financial statements that give a true and fair view (or are presented fairly in all material
respects) in accordance with the applicable financial reporting framework and managing risks that may give rise to
material misstatements in those financial statements. Such controls reduce but do not eliminate the risks of misstatement.
An auditor conducting an audit in accordance with NSAs obtains reasonable assurance that the financial statements taken
as a whole are free from material misstatement, whether caused by fraud or error. An auditor cannot obtain absolute
assurance that material misstatements in the financial statements will be detected because of such factors as the use of
judgment, the use of testing, the inherent limitations of internal control and the fact that much of the audit evidence
available to the auditor is persuasive rather than conclusive in nature.
The primary objective of an auditor is to express an opinion on the financial statements. However, the auditor while
conducting the audit is required to consider the risk of material misstatements in the financial statements resulting from
fraud or error. An audit conducted in accordance with the auditing standards generally accepted in Nepal is designed to
provide reasonable assurance that the financial statements taken as a whole are free from material misstatement, whether
caused by fraud or error. The fact that an audit is carried out may act as a deterrent, but the auditor is not and cannot be
held responsible for the prevention of fraud and error.
The auditor's opinion on the financial statements is based on the concept of obtaining reasonable assurance; hence, in
an audit, the auditor does not guarantee that material misstatements, whether from fraud or error, will be detected.

b) Materiality is an important consideration for an auditor to evaluate whether the financial statements reflect a true
and fair or not. The auditor should consider materiality and its relationship with audit risk when conducting an audit. The
Nepal Accounting Standards Board's "Framework for the Preparation and Presentation of Financial Statements" defines
Materiality in the following terms: "Information is material if its omission or misstatement could influence the economic
decisions of users taken on the basis of the financial statements. Materiality depends on the size of the item or error
judged in the particular circumstances of its omission or misstatement. Thus, materiality provides a threshold or cut-off
point rather than being a primary qualitative characteristic which information must have if it is to be useful."
In designing the audit plan, the auditor establishes an acceptable materiality level so as to detect quantitatively material
misstatements. However, both the amount (quantity) and nature (quality) of misstatements need to be considered.
Examples of qualitative misstatements would be the inadequate or improper description of an accounting policy when it is
likely that a user of the financial statements would be misled by the description, and failure to disclose the breach of
regulatory requirements when it is likely that the consequent imposition of regulatory restrictions will significantly impair
operating capability.
The auditor should consider the possibility of misstatements of relatively small amounts that, cumulatively, could have a
material effect on the financial statements. For example, an error in a month end procedure could be an indication of a
potential material misstatement if that error is repeated each month.
The auditor should also consider materiality at both the overall financial statement level and in relation to individual
account balances, classes of transactions and disclosures. Materiality may be influenced by considerations such as legal
and regulatory requirements and considerations relating to individual financial statement account balances and
relationships. This process may result in different materiality levels depending on the aspect of the financial statements
being considered.
While determining the nature, timing and extent of audit procedures; and evaluating the effect of misstatements, the
auditor should consider the materiality. When planning the audit, the auditor should consider what would make the
financial statements materially misstated. The auditor's assessment of materiality, related to specific account balances and
classes of transactions, helps the auditor decide such questions as what items to examine and whether to use sampling and
analytical procedures. This enables the auditor to select audit procedures that, in combination, can be expected to reduce
audit risk to an acceptably low level. There is an inverse relationship between materiality and the level of audit risk, that
is, the higher the materiality level, the lower the audit risk and vice versa.
The auditor in addition to exercising professional judgment should consider any legislation or regulation which may
impact that assessment while assessing materiality in the case of the public sector. In the public sector, materiality is also
based on the “context and nature” of an item and includes, for example, sensitivity as well as value. Sensitivity covers a
variety of matters such as compliance with authorities, legislative concern or public interest.

3. Give your comments on the following: (35=15)


a) You have attended physical verification of Supreme Garments Limited and noted no discrepancies between the
physical quantity and the Store Register for the year ending 15 July, 2010. The management provides a statement of value of
stocks but refuses to provide with the cost sheet and calculations regarding how the value has been arrived. The company
discloses that the value of stocks is as “certified by the management” in the Financial Statement. The management is ready
to disclose the same in the Management Representation letter.
(18)

b) The Critical Pollution Extinction Company Limited procured a pollution controlling machine for which the
government has 50% rebate in customs duty upon the precondition that the machine should be used for at least 5 years.
During the course of audit you found that the company has credited 50% rebate provided to income for the year by
disclosing the same in the Notes to Accounts in the Financial Statement. What is your opinion as regards the accounting
treatment by the company?

c) Mr. Yadav, a Chartered Accountant in practice enters into an agreement with Mr. Puspa Adhikari, an individual
who has passed one group of CA final 2 years back. The agreement provides that Mr. Puspa Adhikari shall work in all
professional assignments of Mr. Yadav and shall receive 15 percent of fee received from such assignments as remuneration.
Explain whether the agreement is in order as per Code of Ethics or any other relevant reference? If remuneration is based on
cash flow, does it violate the provisions of the Code of Ethics?

Answer No.3
a) As per NSA-500 “Audit Evidence”, Audit evidence regarding an assertion of existence of inventory will not
compensate for failure to obtain assertion regarding valuation. The auditor should satisfy himself that the valuation of
inventories is in accordance with the Nepal Accounting Standard -02 “Inventories”. The auditor should examine the
methods of applying the basis of inventory valuation which include examination of stock sheets, costing records and
treatment of overhead expenses as a part of cost of inventories. In the given case, it will be construed as limitation on the
scope of auditors. Accordingly, the auditor will have to issue a qualified opinion.
Certification by Management of Supreme Garments Ltd. cannot be taken as conclusive evidence and it cannot relieve the
duty of auditors when there are other procedures and means of gathering audit evidence. Just because management had
owned responsibility for the correctness of its valuation of Stock/inventory, the auditor cannot shirk his responsibility.
This is negligence on his part if he relies on the management representation without assessing the corroborative available
evidences.

b) As mentioned in Para 8 of NAS 10 Accounting for Government Grants and Disclosure of Government
Assistance, a government grant is not recognized until there is reasonable assurance that the entity will comply with the
conditions attaching to it, and that the grant will be received. Receipt of a grant does not of itself provide conclusive
evidence that the conditions attaching to the grant have been or will be fulfilled. Government grants shall be recognized as
income over the periods necessary to match them with the related costs which they are intended to compensate, on a
systematic basis. They shall not be credited directly to shareholders‟ interests. It is fundamental to the income approach
that government grants be recognized as income on a systematic and rational basis over the periods necessary to match
them with the related costs. Income recognition of government grants on a receipts basis is not in accordance with the
accrual accounting assumption. Government grants related to assets, including non- monetary grants at fair value, shall be
presented in the balance sheet either by setting up the grant as deferred income or by deducting the grant in arriving at the
carrying amount of the asset. Hence the NAS has provided two acceptable alternative methods of presentation in financial
statements of grants related to assets. According to the first method, the grant amount is accounted for as deferred income
which is recognized as income on a systematic and rational basis over the useful life of the asset. Under the other
alternative method, government grant is deducted in arriving at the carrying cost of the asset and the grant is recognized as
income over the depreciable asset by way of a reduced depreciation charge.
Hence, as mentioned here above, the treatment to credit whole of the credit rebate amount in the year of purchase of the
machine by Critical Pollution Extinction Company is not appropriate since it is bound by the precondition that the
machine should be used for at least 5 years. The 50% rebate received on customs duty should be credited to income for at
least over a period of 5 years.

c) As per section 34(3) of Nepal Chartered Accountants Act, 1997, one shall not share the auditing fees or
remuneration or distribute as profit with any person other than a member of the institute and shall not pay any
commission, brokerage, etc. out of the professional fees earned to any person or member.
In the above case also, Mr. Yadav a Chartered Accountant in practice enter into agreement to share auditing fees.
Accordingly, Mr. Yadav would be held guilty of professional misconduct since he agreed to share the fees to his
employee.
Even if remuneration were based on cash flow, it would tantamount to be based on fees received on cash and accordingly
Mr. Yadav would be held guilty of professional misconduct.

4. Answer the following: (35=15)


a) Babarmahal Trading Ltd (BTL) is a wholesaler of Chinese goods and all accounting information is stored on BTL‟s
computer. You are the audit senior in charge of the audit of the receivables balance. Explain the audit procedures that
should be carried out on the receivables balance at BTL.
b) Proper audit is not possible without adequate knowledge of client‟s business. Explain it.
c) Define the concept of internal control and explain its inherent limitations.

Answer No.4
(19)

a) Procedures for verification of receivable balance of BTL are prescribed below:


 Cast the receivables ledger to ensure it agrees with the total on the receivables control account.
 Compare the balance on each receivable account with its credit limit to ensure this has not been exceeded.
 Review the balances in the receivables ledger to ensure no balance exceeds total sales to that customer.
 Calculate receivables days for each month end to monitor control of receivables over the year.
 Stratify receivables balances to show all material items and select appropriate sample for testing.
 Produce an aged receivables analysis to assist with the identification of irrecoverable receivables.
 Obtaining the external confirmations as per NSA 505 and evaluating whether the results of the external confirmation
process together with the results from any other audit procedures performed, provide sufficient appropriate audit
evidence regarding the assertion being audited. In conducting this evaluation the auditor considers the guidance
provided by NSA 330 and NSA 530, “Audit Sampling and Other Selective Procedures.”

b) Obtaining an understanding of the entity and its environment is an essential aspect of performing an audit in
accordance with Nepal Standard on Auditing . In particular, that understanding establishes a frame of reference within
which the auditor plans the audit and exercises professional judgment about assessing risks of material misstatement of the
financial statements and responding to those risks throughout the audit, for example when:
 Establishing materiality and evaluating whether the judgment about materiality remains appropriate as the audit
progresses;
 Considering the appropriateness of the selection and application of accounting policies, and the adequacy of financial
statement disclosures;
 Identifying areas where special audit consideration may be necessary, for example, related party transactions, the
appropriateness of management‟s use of the going concern assumption, or considering the business purpose of
transactions;
 Developing expectations for use when performing analytical procedures;
 Designing and performing further audit procedures to reduce audit risk to an acceptably low level; and
 Evaluating the sufficiency and appropriateness of audit evidence obtained, such as the appropriateness of assumptions
and of management‟s oral and written representations.
The auditor uses professional judgment to determine the extent of the understanding required of the entity and its
environment, including its internal control. The auditor‟s primary consideration is whether the understanding that has been
obtained is sufficient to assess the risks of material misstatement of the financial statements and to design and perform
further audit procedures. The depth of the overall understanding that is required by the auditor in performing the audit is less
than that possessed by management in managing the entity.

c) The concept of internal control may be defined as the plan of organization and all the methods and procedures
adopted by the management of an entity to assist in achieving management‟s objectives of ensuring the orderly and efficient
conduct of its business, including adherence to management policies, the safeguarding of assets, prevention and detection of
fraud and error, the accuracy and completeness of the accounting records, and the timely preparation of reliable financial
information. The system of internal control extends beyond those matters which relate directly to the functions of the
accounting system and comprises control environment and control procedures. Internal control is an essential prerequisite
for efficient and effective management of any organization. It is thus, a primary responsibility of every management to
establish and maintain an adequate system of internal control appropriate to the size and nature of the business of the entity.
An internal control system can provide only reasonable, not absolute, assurance that the management‟s objectives in
establishing the system are achieved. This is because there are some inherent limitations of internal control. These
limitations are mentioned hereunder:
 Controls have to be cost effective. Hence, some control mechanisms may not have been implemented merely because
they are not cost-effective.
 Most control tools are directed at transactions of a usual nature. Therefore, transactions of unusual nature might have
been escaped from such controls.
 The human error potentiality prevails everywhere, even in the control systems.
 Any system of control has its limitations in preventing frauds through collusion between two or more persons.
 Controls may not change with the pace of changes in conditions.
 Management itself may manipulate transactions or accounting estimates.
 A member of management may himself override the control system.

5. Answer the following: (35=15)

a) Nepal Standards on Auditing relating to "Management Representation" provides guidance on the use of management
representation as audit evidence. Enumerate five items that could be included in a management representation letter.
b) Briefly explain five methods of sampling.
c) What are the principal contents of audit engagement letter?

Answer No. 5
(20)

a) Following matters form part of the management representation letter as per NSA 580 „Management
Representations‟:
 It is management's responsibility for the fair presentation of the financial statements in accordance with Nepal
Accounting Standards or relevant practices.
 No irregularities involving management or employees that could have a material effect on the financial statements
 All books of account and supporting documentation have been made available to the auditors
 Information and disclosures with reference to related parties is complete
 Financial statements are free from material misstatements including omissions
 No non-compliance with any statute or regulatory authority requirements.
 No plans that will materially alter the carrying value or classification of assets or liabilities in the financial statements
 No events, unless already disclosed, after the end of the reporting period that needs disclosure in the financial
statements.

b) Methods of sampling is presented below:


Haphazard Sampling
Haphazard sampling is a technique adopted by the auditor where the sample does not follow a structured technique.
Haphazard sampling is not appropriate when using statistical sampling and the auditor should always ensure that haphazard
sampling is not „doctored‟ in such a way that it deliberately avoids sampling items which, for example, are difficult to
locate. All items in the population should stand a chance of being sampled.
Stratified Sampling
This is a technique where the auditor will split items in a sample into their various strata‟s. For example, in a payroll sample
the auditor might split the sample between full-time males, full-time females, part-time males and part-time females and
work out the percentage of the strata in the population (the population being the total amount that makes up a figure). For
example if 30% of the population are full-time males, 40% full-time females, 20% part-time males and 10% part-time
females, then the sample will consist of 30% full-time males, 40% full-time females etc.
Systematic Sampling
Often referred to as „interval‟ sampling this is where the auditor will take the number of sampling units in the population
and divide this into the sample size to give a sampling interval. For example in a sales invoice sample where the sampling
interval is 20, then the auditor will determine a starting point for sampling and sample every 20th sales invoice thereafter.
Block Sampling
Block sampling is a technique where the auditor applies procedures to such items that all occur in the same block of time or
sequence. For example testing amounts received from customers in the month of September. Alternatively, a „block‟ of
remittance advices received in September would be tested in their entirety. It is to be noted that block sampling should be
used with caution because valid references cannot be made beyond the period or block examined. Where the auditor does
use block sampling, then many blocks should be selected to help minimise sampling risk.
Judgement
Auditors can use their judgement in selecting items for sampling. There are three basic issues which determine which items
are selected:
 The value of items
 The relative risk (items prone to error should be given special attention)
 The representativeness (the sample should be representative of the population)

c) The form and content of audit engagement letters may vary for each client, but they would generally include
reference to:
 The objective of the audit of financial statements;
 Management‟s responsibility for the financial statements;
 The scope of the audit, including reference to applicable legislation, regulations, or pronouncements of professional
bodies to which the auditor adheres;
 The form of any reports or other communication of results of the engagement;
 The fact that because of the test nature and other inherent limitations of an audit, together with the inherent limitations
of internal control, there is an unavoidable risk that even some material misstatement may remain undiscovered; and
 Unrestricted access to whatever records, documentation and other information requested in connection with the audit.
 Management‟s responsibility for establishing and maintaining effective internal control.

6. Write short notes on the following: (25=10)
a) Disadvantages of the use of an audit program.
b) Flow Chart

Answer No. 6
a) The possible disadvantages of the use of an audit program are:
i. The work may become mechanical and particular parts of the programme may be carried out without any
understanding of the object of such parts in the whole audit scheme.
(21)

ii. The programme often tends to become rigid and inflexible following set grooves; the business may change in its
operation of conduct, but the old programme may still be carried on. Changes in staff or internal control may render
precaution necessary at points different from those originally decided upon.
iii. Inefficient assistants may take shelter behind the programme i.e. defend deficiencies in their work on the ground that no
instruction in the matter is contained therein.
iv. A hard & fast audit program may kill the initiative of efficient and enterprising assistants.
v. Preparation of audit program without preliminary depth assessment of clients business and without considering scope of
work to be carried out by auditor will not provide objectivity of the audit work.

c) Flow Chart is a graphical presentation of all the processes within an organization. It gives bird`s eye view of the system
and the flow of transactions that is of particular interest for the auditors. It facilitates the auditors in accumulation of the
information necessary for the proper review and evaluation of internal controls implemented within an organization. It
is considered to be the most concise way of recording the auditor`s review of the system. It minimizes the amount of
narrative explanation and thereby achieves a consideration or presentation not possible in any other form.

7. Justify with reason, whether following statement is true or false. (25=10)


a) Reducing assurance engagement risk to zero is very rare.
b) Balance confirmation from debtors/creditors can only be obtained at the end of the financial year.

Answer No. 7
a) The statement is true. Reducing assurance engagement risk to zero is very rare because of the following reasons:
 The use of selective testing;
 The inherent limitations of internal control;
 The fact that much of the evidence available to practitioner is persuasive rather than conclusive; and
 The use of judgment in gathering and evaluating evidence and framing conclusions based on that evidence.

b) The statement is false. Direct confirmation of balances from debtors/creditors in respect of balances standing in
their accounts at end of the financial year end is perhaps the best method of ascertaining whether the balances are genuine
accurately stated and undisputed where internal control system is relatively weak. The conformation date, methods of
obtaining confirmation etc. are to be determined by the auditor. Debtors/creditors may be requested to confirm balance
either as at the date of balance sheet or any other selected date which reasonably close to the date of balance sheet. The
date should be settled by the auditor in consultation with the entity. Therefore, it is not necessary that balances of
debtors/creditors should be obtained and verified at the end of financial year.
The Institute of Chartered Accountants of Nepal
Suggested Answers of Corporate and Other Laws
CAP II Examination – June 2011

1.
a) The Registrar of Companies issued a certificate of incorporation on 8th January, 2011. However, by mistake, the
certificate was dated "5th January 2011" and the allotment of shares was made on 7 th January 2011. Can the allotment be
declared void on the ground that it was made before the company was incorporated? Explain with reference to the
provisions of the Companies Act, 1956? 5

b) There are various legal relations between companies based on shareholdings. Based on relevant provisions of
Companies Act, 1956 in reference, state the relations in the following cases of:
i) Gaumati Company Ltd. incorporated in 1950 have equal equity and preference shares in its share capital. Both
shareholders are equal voting rights in the general meeting of the company. Fedi Lal Company Ltd. is holding 100% of
preference share and 20% of equity shares. Does this relation create holding subsidiary relationships? 5

ii) Binayak Company Ltd. is subsidiary of Amar Company Ltd., Chadani Company Ltd. is subsidiary of Binayak
Company Ltd. Damodaran Company Ltd. is Subsidiary of Chadani Company Ltd. What is the relation of Amar Company
Ltd. and Damodaran Company Ltd? 5

c) CG Company Ltd. had issued a cheque for Rs. 30.00 lacs to Mr. X in full discharge of the debt owed by the company to
Mr. X. The cheque was presented to the Bank for payment however it was dishonored for insufficiency of funds. Advice
Mr. X regarding the provisions of the Negotiable Instruments Act, 1881 for taking a legal recourse. 5

d) Enumerate the circumstances in which a company may be wound up by the Tribunal as per provisions of the
Companies Act, 1956? 5

Answer No 1
a) The allotment of shares is valid. Section 35 of the Companies Act, 1956 provides that a certificate of incorporation
issued by the Registrar in respect of any association shall be conclusive evidence of the fact that all the requirements of the
Act have been complied with respect of registration and matters precedent and incidental thereto, and that the association is
a company authorized to be registered and duly registered under the Act.

The Certificate of Incorporation is conclusive evidence as to all administrative acts relating to incorporation and as to the
date of incorporation. In Jubilee Cotton Mills v. Lewis (1924) A.C.958, the Registrar issued on 8 th January a certificate
dated 5th January. It was held that an allotment of shares made on 7 th January could not be declared void on the ground that
it was made before the company was incorporated. The certificate is not, however, conclusive evidence of that all the
objects of the company, as set out in the memorandum, are legal. (Bowen v. Secular Society (1917) A.C. 406, 432)

b) (i) Companies established before enactment of Indian Companies Act, 1956 are called existing companies for the
purpose of Companies Act, 1956. According to provision of Sec. 4(b)(i), existing company having provision that preference
share has voting right in general meeting as equity shares, holding-subsidiary relation to be counted based on combined
voting powers. In this case, Fedi Lal Company Ltd. is holding 100% of preference share and 20% of equity shares. In the
absence of further information, assuming preference shares and equity shares are in equal weight, Fedi Lal Company Ltd
has 60% voting rights in general meeting, so it is holding company for Gaumati Company Ltd.
(ii) Subsidiary of Subsidiary company is subsidiary for the first holding company. According to provision of Sec. 4(c) and
illustration thereto of Companies Act, 1956, Amar Company Ltd. and other relation is as follows:
Amar Company Ltd. Holding of all companies
Subsidiary
Binayak Company Ltd. Holding of Chadani and Damodaran companies
Subsidiary
Chadani Company Ltd. Holding of Damodaran companies
Subsidiary
Damodaran Company Ltd.

c) According to the Section 138 of the Negotiable Instruments Act, 1881,where any cheque drawn by a person on an
account maintained by him with a banker for payment of any amount of money to another person from out of that account
for the discharge, in whole or in part, of any debt or other liability, is returned by the bank unpaid either because of the
amount of money standing to the credit of that account is insufficient to honour the cheque or that it exceeds the amount
arranged to be paid from that account by an agreement made with that bank, such person shall be deemed to have committed
an offence and shall, without prejudice. to any other provision of this Act, be punished with imprisonment for a term which
may extend to two year, or with fine which may extend to twice the amount of the cheque, or with both:

NMG P.T.O.
(23)

Provided that nothing contained in this section shall apply unless-


(a) the cheque has been, presented to the bank within a period of six months from the date on which it is drawn or within the
period of its validity, whichever is earlier;
(b) the payee or the holder in due course of the cheque as the case may be, makes a demand for the payment of the said
amount of money by giving a notice, in writing, to the drawer of the cheque, within 30 days of the receipt of information by
him from the bank regarding the return of the cheque as unpaid; and
(c) the drawer of such cheque fails to make the payment of the said amount of money to the payee or, as the case may be, to
the holder in due course of the cheque, within fifteen days of the receipt of the said notice.
Explanation.-For the purposes of this section, "debt or other liability" means a legally enforceable debt or other
liability.

As per provisions of Section 138 of the Negotiable Instruments Act mentioned above dishonour of a cheque for non
availability of funds is an offence for which Mr. X can sue the company subject to compliance of the conditions mentioned
therein. Hence Mr. X should comply with the conditions mentioned in the proviso to Section 138 and if the cheque is
dishonoured even thereafter then he can initiate legal remedies.

d) A company may be wound up by the Tribunal under the following circumstances ( Section 433)
(a) if the company has, by special resolution, resolved that the company be wound up by the tribunal;
(b) if default is made in delivering the statutory report to the Registrar or in holding the statutory meeting;
(c) if the company does not commence its business within a year from its incorporation, or suspends its business for a
whole year;
(d) if the number of members is reduced, in the case of a public company, below seven, and in the case of a private
company, below two;
(e) if the company is unable to pay its debts;
(f) if the tribunal is of the opinion that it is just and equitable that the company should be wound up;
(g) if the company has made a default in filing with the registrar its balance sheet and profit and loss account or annual
return for any five consecutive financials years;
(h) if the company has acted against the interests of the sovereignty and integrity of India, the security of the State, friendly
relations with foreign States, public order, decency or morality;
(i) if the Tribunal is of the opinion that the company should be wound up under the circumstances specified in section 424
G;
Provided that the Tribunal shall make an order for winding up of a company under clause (h) on application made by the
Central Government or a State Government.

Part : "B"

2.
a) M/s Duggar Enterprises has entered into a contract for carriage of food items from Nepalgunj to Kathmandu, with
United Transport Ltd. This arrangement has been continuing for the the last three years. During Kartik 2067 due to Tihar
festival, demand for fresh fruit emerges in Kathmandu as a result the unit located at Neplagunj sends one truckload of fresh
fruit worth Rs. 10.00 lacs in the truck meant for supplying foodgrains using the same contract. The driver of the truck while
being midway goes and visits his sister, living in the vicinity of Butwal, on Bhai Tika day. The act of the driver delays the
delivery of goods by two days as a result the whole truckload of fruits gets rotten and damaged? Examine the liability of the
transporter as per the Contract Act, 2056? 8

b) What conduct of worker or employee shall be deemed as misconduct under Labour Act, 2048? Discuss briefly. 7

Answer No.2

a) The liability of a transporter as limited by the Contract Act 2056 is elaborated as follows:

Section 68. Liability of transporter to be limited:


(1) Except when the owner of goods or his/her agent has clearly declared at the time of concluding the contract that the
goods to be transported are worth more than Ten Thousand Rupees or except when otherwise provided for in the contract,
the amount of compensation to be paid by a transporter for any loss or damage to goods transported by him/her shall not
exceed Ten Thousand Rupees.

(2) Notwithstanding anything contained in Sub-section (1), the transporter shall not be held liable for any loss or
damage to gold, sliver, diamond jewels or goods made thereof, precious stone, negotiable instruments, securities, documents
registered by offices, certificates issued by educational and other institutes, coins, bank notes, postal stamps, fish, meat,
fresh fruits and vegetables, insecticide, inflammable materials, petroleum products, precious art pieces, idols, curio goods or
fragile goods, wildlife and pets, handicrafts, arms and ammunition, explosives, radio, television, computer and similar other
goods and their spare parts, machinery and goods specified in prevailing law as those which have to be declared by the
(24)

owner before their transportation, except when the owner or his agent has clearly declared them at the time of signing the
contract or at the time of handing them over for transportation to the transporter.

(3) For the purpose of bearing the risk involved in the transportation of goods mentioned in Sub-section (2), the
transporter might have them insured against the risk through their owner or his/her agent or by him/herself by collecting a
separate fee for the purpose, or take other necessary arrangement to avoid the risk.

Considering the above mentioned provisions the transporter may not be liable for compensation as per Section
68(2) mentioned above.

b) As per section 51 of the Labour Act, 2048, for the purpose of section 50, the following conduct of
the worker or employee shall be deemed as misconduct:
(a) In case of any bodily harm or injury or fetters or detains to the Proprietor, Manager or Employee of the Enterprise with
or without use of arms or injury or causes any violence or destruction or assault within the Enterprise in connection with the
labour dispute or on any other matter;
(b) In case creates or causes to create any stir within the Enterprise with an intention or affecting the production process or
service works of the Enterprise, or prevents the supply of food and water, or connection of telephone and electricity, or
obstructs the entry into or movement within the Enterprise;
(c) In case steals the property of enterprise;
(C1) If commits embezzlement in the transactions of the Enterprise,
(C2) If absents in the Enterprise more than a consecutive period of thirty days without notices.
(d) In case accepts or offers brides;
(e) In case imprisoned on being convicted on a criminal offence Involving moral turpitude.
(f) In case participates or compels any other person to participate
in any authorized strike or in a strike which is declared illegal;
(g) In case strikes without fulfilling the legal requirements or intentionally slow down the work against interests of the
Enterprise;
(h) In case intentionally destroys any property of the Enterprise, or causes damage thereon or takes and uses it outside the
Enterprise or gives its use to unauthorized person without permission of the competent person;
(i) In case intentionally violates intentionally the orders or directives issued under this Act or the Rules made hereunder
frequently, or the Bylaws made by the Enterprise, or misbehaves with the customers of the Enterprise;
(j) In case remains absent from the work frequently without obtaining permission or comes late after the regular time;
(k) If it is certified by a doctor that one has come to the duty after consuming or has consumed alcoholic substances during
the working time.
(l) In case performs any activity with a motive of causing damage to secrecy relating to special technology of the Enterprise,

(m) In case abuses any items which has been kept for the interest, Health and safety of the workers or employees or causes
damage to them intentionally;

3.
a) What are the liabilities for the matters referred to in the prospectus as per the Securities Act, 2063? 5

b) What are the circumstances where attendance or voting of shareholders in General meeting is restricted under
Companies Act, 2063? Explain briefly. 5

Answer No.3
a) The liabilities for the matters referred to in the prospectus as laid out in Section 33 of the Securities Act, 2063 are as
follows:

(1) The concerned body corporate and the director signing a prospectus and the expert preparing such a prospectus shall be
severally and collectively liable for the truth of the details and documents underpinning the information set down in the
prospectus submitted to the Board for the purpose of registering securities with the Board and obtaining permission to issue
such securities.
(2) Where any person who subscribes for any securities on the faith of the matters set down in the prospectus subsequently
sustains any loss or damage by the reason that the matters set down in the prospectus have been set down with mala fide
intention or untrue or false statements have been included therein knowingly, the body, director or experts preparing the
prospectus shall be liable to pay compensation for such loss or damage.
Provided that no director shall be liable to pay such compensation if he or she proves that he or she has resigned prior to
making a decision on the matters set down in the prospectus with ulterior motive or knowingly or that he or she did not
know that the prospectus was untrue.
(25)

(3) Where any investor sustains any loss or damage by the reason that the prospectus, information, statements or returns
submitted by a body corporate to the Board, such an investor may make a petition to the concerned District Court for
compensation within thirty five days from the date of knowledge within one year after the making of investment.

b) The companies Act, 2063, provides the following cases where attendance or voting in meeting is restricted (Section 70):
(1) No person shall be entitled to attend and vote in any general meeting, in the capacity of a shareholder, either in person or
by proxy on any discussion to be held in respect of any terms and conditions entered into or to be entered into between
him/herself and the company.
(2) No director or his/her partner or his/her proxy shall be entitled to vote on any discussion to be held at any General
Meeting in respect of the responsibility for any act done or omitted to be done or done wrongfully by him/her or in respect
of his/her own appointment, dismissal, transfer or confirmation, with respect to the provision of, or reduction or increment
in remuneration, allowance or bonus or in respect of any agreement, contract or arrangement regarding his employment or
anything in which his interest or concern is involved.
(3) Any shareholder who has not paid calls on the shares shall not be entitled to attend and vote in the general meeting.
(4) Where any shareholder appoints a director of the concerned company as his proxy pursuant to Sub-section(2) of Section
71, such director shall not be entitled to vote in the general meeting ,as a proxy of any one , on any matter in which his/her
interest or concern is involved or on the matter of his/her appointment.
(5) Notwithstanding anything contained in this Act or the prevailing law, where a bank or financial institution incorporated
under the prevailing law, which institutes legal action against a shareholder, who has borrowed a loan from such bank or
financial institution against the pledge or security of the shares held by him, for his default in repaying the loan, writes to the
concerned company to prevent him from exercising voting right in respect of shares, then the company shall prevent such
shareholder from exercising voting right in respect of the shares held by him/her for a period until he/she repays the loan.

4.
a) What are the procedures for settlement of disputes as per the Foreign Investment & Technology Transfer Act,
2049? 5

b) Write the procedure for amending the Memorandum of Association of a Limited liability company as per
Companies Act, 2063. 5

Answer No 4
a) The procedure for settlement of disputes as provided in Section 7 of the Foreign Investment &
Technology Transfer Act, 2049 are as follows:
1) If any dispute arises between a foreign investor, national investor or the concerned industry, the concerned parties shall
be required to settle the dispute by mutual consultations in the presence of the department.
2) If the dispute cannot be settled in the manner referred to in sub-section (1) above, it shall be settled by arbitration in
accordance with the prevailing arbitration rules of the United Nations Commission on International Trade Law
(UNCITRAL).
3) The arbitration shall be held in Kathmandu. The laws of Nepal shall be applicable in the arbitration.
4) Notwithstanding anything contained in sub-section (1), (2) and (3) above, disputes arising in regards to foreign
investments made in the industries with investments as prescribed may be settled as mentioned in the foreign investment
agreement.

b) According to the provision of Sec. 21 of Company Act, 2063, procedure for amending the
Memorandum of Association is as follows:
 Company at its general meeting with special resolution may change the clauses of MOA.
 The name of the company can be changed upon special resolution and if pre-authorised by the Registrar. Pre-authority
can be taken upon payment of prescribed fees.
 The resolution, so passed shall be laid to the notice of the Registrar within 30 days of date of resolution and the
Registrar shall record it within 7 days of notice so received.
 In case any aggrieved shareholder (holding not less than 5%of the paid up capital) applied to the court, the resolution
shall be suspended till date of court‟s final verdict.

5.
a) Enumerate the disqualification to become a director as listed in the Banks and Financial Institutions Act, 2063?
9

b) State the provisions with respect to contingent contracts as provided in the Contract Act, 2056? 6

Answer No.5
(26)

a) Section 18 of the Banks and Financial Institutions Act, 2063, provides the following provisions with regard
Disqualification of directors:
(1) Any of the following persons shall not be eligible to be elected or nominated to the office of, or to continue to hold the
office of, a director:
(a) Who is below 21 years of age;
(b) Who is of unsound mind or is insane;
(c) Who is a declared insolvent;
(d) Who has been blacklisted in connection with any transaction with any bank or financial institution, and a period of at
least three years has not lapsed after his or her name has been removed from that list;
(e) Who is a director of any bank or financial institution or any financial institution carrying on the transactions relating to
any kind of deposits or an employee serving in such institution or a serving auditor or advisor of any other bank or financial
institution;
(f) Who is a partner in any kind of contract agreement with the concerned bank or financial institution;
(g) Who has acquired membership of the Stock Exchange to act as a securities dealer;
(h) Who has not subscribed the minimum number of shares required to be subscribed to be eligible for appointment as a
director pursuant to the articles of association of the bank or financial institution;
(i) Who is a serving employee of the Government of Nepal, Rastra Bank or bank or financial institution;
Provided that this provision shall not be applicable to the nomination of any person made by the Government of Nepal as a
director of any bank or financial institution of which shares have been purchased by it, or to the nomination of any person
made by any bank or financial institution as a director of any other bank or financial institution of which shares have been
purchased by it;
(j) In the case of a person who has been relieved of or retired from the service of the Rastra Bank after serving in the post of
Governor, Deputy-Governor or Special Class employee, at least one year from the date of such relief or retirement from the
service has not elapsed;
(k) Who, having a liability to pay tax pursuant to the laws in force, has failed to pay the same;
(l) Who is convicted by a court of an offence involving moral turpitude and sentenced in respect thereof, and a period of five
years has not elapsed from the expiry of the sentence;
(n) Who is convicted by a court of an offence of corruption or cheating;
(o) A period of five years has not lapsed after the date on which the Rastra Bank has taken action against him or her for
failure to observe a direction of the Rastra Bank or the date of suspension or dissolution after taking management by the
Rastra Bank under its control.
(2) No person shall continue to hold the office of a director in any of the following circumstances:
(a) If he or she suffers from any disqualification for appointment to the post of director as mentioned in sub- section (1);
(b) If the general meeting passes a resolution to remove him or her from the office of director;
(c) If the resignation tendered by the director is accepted;
(d) If he or she is held by a court to have done any act involving dishonesty or ulterior motive in the activities of the bank or
financial institution;
(e) If he or she does any act prohibited by this Act from being done by a director.

b) The Provisions with regard to Contingent Contracts under Section 12 of the Contract Act, 2056, are as follows:

(1) In case a contract has been concluded for performing or not performing any work if any event happens in the future, the
contract shall not create any liability until such event happens.
(2) In case a contract has been concluded subject to the condition that it shall be deemed to have been concluded in case any
person performs any specified work in the future, no liability shall be deemed to have emerged from that contract if such
person does anything in such a manner as not to perform that work or in such a manner that the work cannot be performed.
(3) In case a contract has been concluded to perform or not to perform any work if any uncertain event does not happen in
future, liability under that contract shall emerge only after the happening of that event becomes impossible.
(4) In case a contract has been concluded with a provision to perform or not to perform any work if any event happens
within a specified period in the future, the contract shall be deemed to have become invalid after the happening of that event
becomes impossible within the specified period or after the expiry of that period.
(5) In case a contract has been concluded with a provision to perform or not to perform any work if any event does not
happen within a specified period in the future, liability under such contract shall emerge if that event does not happen within
that period or if it becomes certain that the event will not happen within that period.

6.
a) What types of establishment require to pay bonus? Can the punishment of establishment for any offence under Bonus
Act, 1974 be inflicted on the General Manager? 5

b) State the composition of council of Institute of Chartered Accountants‟ of Nepal and their tenure of office. 5

Answer No.6
(27)

a) Establishment which makes Profits is eligible to Pay Bonus (Section 5). The other provisions are as
follows
1. Every establishment which makes a profit shall allocate an amount equal to ten percent of the net profit made during a
fiscal year for payment of bonus to its employees.
2. The following amount also shall be debited from the net income Tax Act while determining the net profit made during
any fiscal year by an establishment under Sub-Section (1).
a. Amount set aside for providing quarters to employees under Sub-Section (1) or Section 41 of the Labor
Act 2048.
b. Deleted …….
c. Amount of bonus paid in excess under Sub-Section (3) of Section 11.
3. Notwithstanding anything contained in Sub-Section (1), the percentage and other matters relating to bonus payable by
an establishment owned by Government shall be as decided by Government.

Responsibility of General Manager (Section 23)


In case any establishment is to be punished for any offense punishable under this act, such punishment shall be inflicted on
the person who was functioning in the capacity of its general Manager at the time when such offense was committed.
Provided that a General Manager who is able to prove that such offense was committed without his knowledge or consent
shall not be liable to punishment under this section.

b) According to Sec. 7 of Institute of Chartered Accountants of Nepal Act, 2053, the composition of Council shall be as
follows:
(a) Ten persons elected by and amongst Chartered Accountant members -Member
(b) Four persons elected by and amongst Registered Auditors -Member
(c) Three persons nominated by Government of Nepal, upon the recommendation of the Auditor General, from amongst
the persons well experienced in the field of accounting profession. -Member
The Council members shall elect a President and a Vice-President from the Fellow Chartered Accountants (F.C.A) Council
members.
The term of office of the President and the Vice-President shall be of one year and upon expiry of the term of office, they
shall be eligible to be elected for one more term.
The term of office of the Council members shall be of three years and upon expiry of the term of office, they shall be
eligible to be re-elected or re-nominated.

7. Write short notes on: (3×5=15)

a) Methods of Audit under Audit Act, 2048


b) Payment of Insurance claims after the cancellation of the Insurer
c) Negotiation

Answer No 7
a) Under Section 3 of Audit Act 2048.
1. The auditor general may conduct final audit of the financial activities and other activities relating thereto of the offices,
bodies or organizations under its jurisdiction, either in detail or sporadically or in samples, prescribe scope, methodology
and timing of audit and present the facts obtained there from make critical comments thereon and submit its reports.
2. The auditor general may, if it deems necessary in course of audit, exercise the following powers

a) To check at any time the status of the program and project being operated under the grants obtained by Nepal
Government and examine documents relating to accounts;
b) To require contractors of government contracts to produce relevant documents or other evidence relating to the contract,
which are supposed to be in his possession;
c) To hire services of any expert on the task of audit and, if necessary, engage him under contract with reasonable
remuneration.

b) According to Sec. 16 of Insurance Act, 2052 regarding Payment of Insurance Claims After the
Cancellation of the Insurer, the Insurer, dissolved by the cause of the cancellation of its registration
pursuant to Section 13, shall refund the amount received by it for Insurance to the person, organization
or the Board, within the period and method specified by the Board. It shall refund the principal amount
along with bonus as specified by the Board in the case of Life Insurance and it shall refund the
principal amount as specified by the Board on a proportional basis in the case of Non-Life Insurance.
(28)

c) Negotiation means the act of transfer of Negotiable Instruments to any person so as to constitute
that person the holder thereof. When the instrument is transferred either by delivery or by endorsement
and delivery so as to constitute the transferee as a holder thereof, the instrument is said to be negotiated.
An instrument can be negotiated in following ways-
i) Delivery: The function of making, acceptance or endorsement of a Negotiable Instrument shall be regarded to be
completed only after its delivery.
ii) Negotiation by Delivery: A Negotiable instrument payable to bearer is negotiable by delivery thereof. If the Negotiable
Instrument is payable to the person written on the instrument or to one ordered by him/her, such instrument should be
endorsed by the holder.
iii) Effect of Endorsement: The right shall be transferred after the endorsement of a Negotiable Instrument followed by
delivery. The holder may give such instrument to any other by endorsing it or receive payment by himself.
The Institute of Chartered Accountants of Nepal
Suggested Answers of Cost and Management Accounting
CAP II Examination – June 2011

1) Alpha Limited has prepared the following budget estimates for the coming year:
Product A Product B
Sales (in units) 6,000 16,000
Rs./Unit Rs./Unit
Selling Price 400 640
Direct Materials 120 220
Direct Wages @ Rs. 10 per hour 80 120
Variable Overheads 40 60
Fixed Overhead 80 120
Total Cost 320 520
Profit 80 120
After finalization of the above budget estimates, it is observed that 1/3rd of the production capacity are still idle. In order to
improve the performance, the following proposals are under consideration:

a) Product A will be discontinued and the capacity so released will be used for product B. The selling price of product B
will, however, have to be reduced by Rs. 20 per unit in order to increase the volume of sales.
b) Product B will be discontinued and the capacity so released will be diverted to the production of product C. The
particulars relating to per unit of product C are as under:
Selling Price Rs. 520 Direct Wages Rs. 100
Direct Materials Rs. 150 Variable Overheads Rs. 50
c) The idle capacity will be utilised for meeting an export demand for product D. The particulars relating to per unit of
product D are as under:
Selling Price Rs. 720 Direct Wages Rs. 200
Direct Materials Rs. 400 Variable Overheads Rs. 100
d) The idle capacity will be hired out by fixing a price in such a way that the same rate of profit per direct labour hour as
obtained in the budget estimates is achieved.

Required: (4+15+1=20)
i) Prepare a statement showing the profitability of the products A & B as envisaged in the budget estimates.
ii) Evaluate each of the above four proposals separately showing the profitability under each proposal.
iii) Recommend most profitable proposal for Alpha Limited.

Answer:
Alpha Ltd.
(i) Budgeted Profitability Statement
Product A Product B
Sales units 6,000 16,000
Rs./Unit Rs./Unit
Sales price per unit 400 640
Variable cost per unit:
Direct Material 120 220
Direct Wages 80 120
Variable Overheads 40 60
240 400
Contribution per unit 160 240
Total Contribution A = Rs. 160 × 6,000 Rs. 960,000
B = Rs. 240 × 16,000 Rs. 3,840,000
Rs. 4,800,000
Less: Fixed Cost Rs. 2,400,000
Profit Rs. 2,400,000

(ii) Evaluation of proposals


Proposal (a): Discontinue Product A and use the capacity for Product B

Spare capacity because of discontinuing Product A (8 × 6000) 48,000 hours


Additional units of Product B produced = 48,000/12 4,000 units
Total production of Product B = (16,000 + 4,000) units 20,000 units

NMG P.T.O.
(30)

Statement of profitability
Sales unit 20,000
Rs./Unit
Selling Price per unit Rs. (640 – 20) 620
Variable cost: Direct Materials 220
Direct Wages 120
Variable Overheads 60
400
Contribution per unit 220
Total Contribution (Rs. 220 × 20,000) Rs. 4,400,000
Less: Fixed Cost Rs. 2,400,000
Profit Rs. 2,000,000

Proposal (b): Discontinue Product B and use the capacity for new Product C

Spare capacity because of discontinuing Product B (12 × 16,000) 192,000 hrs


Units of Product C produced = 192,000/(100/10) 19,200 units

Statement of Profitability
Product A Product C
Sales units 6,000 19,200
Rs./Unit Rs./Unit
Sales price per unit 400 520
Variable cost per unit:
Direct Material 120 150
Direct Wages 80 100
Variable Overheads 40 50
240 300
Contribution per unit 160 220
Total Contribution A = Rs. 160 × 6,000 Rs. 960,000
C = Rs. 220 × 19,200 Rs. 4,224,000
Rs. 5,184,000
Less: Fixed Cost Rs. 2,400,000
Profit Rs. 2,784,000

Proposal (c): Use idle capacity for export demand of Product D

Idle production capacity (labour hours) 120,000 hrs


Units of Product D produced = 120,000/(200/10) 6,000 units

Statement of Profitability
Product A Product B Product D
Sales units 6,000 16,000 6,000
Rs./Unit Rs./Unit Rs./Unit
Sales price per unit 400 640 720
Variable cost per unit:
Direct Material 120 220 400
Direct Wages 80 120 200
Variable Overheads 40 60 100
240 400 700
Contribution per unit 160 240 20
Total Contribution A = Rs. 160 × 6,000 Rs. 960,000
B = Rs. 240 × 16,000 Rs. 3,840,000
D = Rs. 20 × 6,000 Rs. 120,000
Rs. 4,920,000
Less: Fixed Cost Rs. 2,400,000
Profit Rs. 2,520,000

Proposal (d): Hire out idle capacity


Statement of Profitability
Rs.
(31)

Profit as per Budget Estimates (I) 2,400,000


Budgeted Direct Labour Hours 240,000
Profit per Direct Labour Hour 10
Idle Capacity (Direct Labour Hour) 120,000
Additional profit by hiring out the idle capacity (II) 1,200,000
Total Profit [(I) + (II)] 3,600,000

(iii) Recommendation:
Since proposal (d) gives highest profit i.e. Rs. 3,600,000 compared to the all other proposals, it is recommended to go for
Proposal (d).

Working Notes:
(i) Idle Production Capacity (Labour Hour)
Product A Product B Total
Direct Labour Hour per unit 80/10 = 8 120/10 = 12
Number of units 6,000 16,000
Direct Labour Hours utilised 48,000 192,000 240,000
The above utilised hours is only 2/3rd. Hence Total Capacity
hours =240000/ 2/3rd 360,000
Idle Capacity 120,000

(ii) Fixed Cost


Rs.
Product A = Rs. 80 × 6,000 480,000
Product B = Rs. 120 × 16,000 1,920,000
Total 2,400,000

2)
a) Your company operates for 300 days a year on average. It is facing severe problem of electric power cut in its day to
day operation. The electricity supply is not available for nearly 4 hours per day in average during total working hours of 7
hours per day for whole the year. This situation is expected to prevail for nearly five more years. To cope with this situation,
you are considering the alternative sources of power generation, i.e. 140 KVA Generator Set and you desire to know the
cost per unit of electricity generated.

The following estimations are available:


i) Number of units to be generated per month is 10,000.
ii) Cost of Gen Set with installation charges is Rs.5 million. It is to be fully depreciated within 5 years, after that period, it
can be disposed off for Rs.250,000.
iii) Regular cleaning and repair and maintenance cost per month is Rs.20,000.
iv) The Gen Set will consume 22 Ltrs. of diesel per hour. The cost of diesel per liter is Rs.65. Other fuel charge is Rs.65
per hour.
v) Two staffs are directly involved in operation and maintenance of the Gen Set, who are paid salary at the rate of
Rs.10,000 per month each.
vi) Share of administrative charges is Rs.10,000 per month.
5

b) A Company produces two joint products P and Q in 70 : 30 ratio from basic raw materials in department A. The input
output ratio of department A is 100 : 85. Product P can be sold at the split of stage or can be processed further at department
B and sold as product AR. The input output ratio is 100 : 90 of department B. The department B is created to process
product A only and to make it product AR.
The selling prices per kg. are as under:
Product P Rs. 85
Product Q Rs. 290
Product AR Rs. 115
The production will be taken up in the next month.
Raw Materials 8,00,000 kgs.
Purchase price Rs. 80 per kg.

Monthly expenses:
Dept. A Dept. B
Rs. in lakhs Rs. in lakhs
(32)

Direct Materials 35.00 5.00


Direct Labour 30.00 9.00
Variable overheads 45.00 18.00
Fixed overheads _40.00 _32.00
Total 150.00 64.00

Selling Expenses:
Rs. in lakhs
Product P 24.60
Product Q 21.60
Product AR 16.80

Required: (4+4=8)
i) Prepare a statement showing the apportionment of joint costs.
ii) State whether it is advisable to produce product AR or not.

c) A telecom company in Nepal has total GSM prepaid active subscriber base of around 5 million. Its Average Revenue
Per User (ARPU) is Rs.238.50 per month. Assuming that its variable cost and profit are 35% and 40% respectively of total
revenue in this segment of operation, you are required to calculate the number of subscriber to be added to justify the 20%
reduction in average call tariff which is currently Rs.1.50 per minute. Also assume that the proposed reduction in tariff will
increase call duration by 20%. 7

Answer:

a) Calculation of the cost per unit of electricity generated


Units generated: 10000/ month
SN Particulars Cost per month Cost per unit Working Remarks
(Rs.) (Rs.)
a. Cost of diesel 143,000 14.30 (22 Ltr. x 4 hrs. x 300/ 12 days x Rs.65)
b. Other fuel charges 6,500 0.65 (Rs. 65x 4 hrs. x 300/ 12 days)
c. Repair & maintenance 20,000 2.00
d. Staff salary 20,000 2.00 (Rs.10000 x 2)
e. Depreciation charge 79,167 7.92 (Rs.5,000,000-250000/5/12)
f. Shared administrative charge 10,000 1.00
Total 278,667 27.87

b) Input in Dept. „A‟ 800,000 kgs.


Yield 85%
Therefore output = 85% of 8,00,000 = 6,80,000 kgs.
Ratio of output for P and Q = 70 : 30
Product of P = 70% of 6,80,000 = 4,76,000 kgs.
Product of Q = 30% of 6,80,000 = 2,04,000 kgs.

(i) Statement showing apportionment of Joint cost


P Q Total
Product Kgs. 4,76,000 2,04,000
Selling price per kg. Rs. 85.00 290.00
Rs. Lakhs Rs. Lakhs Rs. Lakhs
Sales 404.60 591.60 996.20
Less: Selling expenses __24.60 __21.60 __46.20
Net Sales _380 __570 _950
Ratio 40% 60% 100%

Rs. In lakhs
Raw materials (8,00,000 kgs. ×Rs. 80) 640
Process cost of department „A‟ __150
__790
Apportionment of Joint Cost
(In the ratio of Net Sales i.e. P : Q, 40% : 60%)
joint cost of „P‟ = Rs. 316 lakhs
Joint cost of „Q‟ = Rs. 474 lakhs
(33)

(ii) Profitability statement of further processing of product ‘P’ and converting it into product ‘AR’

Output = 90% of 4,76,000 kgs. = 4,28,400 kgs. Rs. in lakhs


Joint costs (As above) 316.00
Cost of department B 64.00
Selling expenses _16.80

396.80
Sales value (Rs. 115×4,28,400) 492.66
Profit (492.66-396.80) __95.86

If „P‟ is not processed profitability is as under:


Rs. in lakhs
Sales 380.00
Less: joint expenses 316.00
Profit __64.00

Further processing of product „P‟ and converting it into product „AR‟ is beneficial to the company because the profit is
increased by Rs. 31.86 lakhs (95.86-64.00).

c)
Computation Table to find out additional number of subscriber to justify tariff reduction
SN Particulars Total Per User Working Remarks
a. No. of Active Prepaid GSM Subscribers 5,000,000
b. ARPU/ month (Rs.) 238.50
c. Total Revenue/ month (Rs.) 1,192,500,000 (a x b)
d. Variable Expenses/ month (Rs.) 417,375,000 35% of Total Revenue
e. Profit/ month (Rs.) 477,000,000 40% of Total Revenue
f. Fixed Costs/ month (Rs.) 298,125,000 (c-d-e)
g. Contribution/ month (Rs.) 775,125,000 155.03
h. Current call tariff/ minute (Rs.) 1.50
i. Average call duration per user/ month 159 (b/h)
j. New call tariff/ minute (Rs.) 1.2 20% less than current
k. New avg. call duration/ user/ month 191 20% more than current
l. New ARPU/ month (Rs.) 229 (j x k)
m. New contribution/ user/ month (Rs.) 149 65% of New ARPU(100-35)
n. Required profit/ month (Rs.) 477,000,000
o. Required no. of total active subscribers = (298125000+477000000)/ 149 (Fixed Cost+Reqd. Profit)/ CMPU
= 5,208,333
p. No. of additional subscribers required to
justify the 20% tariff reduction 208,333 (o-a)

3)
a) Mega Company has just completed its first year of operations. The unit costs on a normal costing basis are as under:
Rs.
Direct material 4 kg @ Rs.4 16.00
Direct labour 3 hrs @ Rs.18 54.00
Variable overhead 3 hrs @ Rs.4 12.00
Fixed overhead 3 hrs @ Rs.6 _18.00
100.00
Selling and administrative costs:
Variable Rs.20 per unit
Fixed Rs.7,60,000

During the year the company has the following activity:


Units produced 24,000
Units sold 21,500
Unit selling price Rs.168
Direct labour hours worked 72,000
(34)

Actual fixed overhead was Rs.48,000 less than the budgeted fixed overhead. Budgeted variable overhead was
Rs.20,000 less than the actual variable overhead. The company used an expected actual activity level of 72,000 direct
labour hours to compute the predetermine overhead rates.

Required: (4+6+2=12)
i) Calculate under or over absorption of overhead.
ii) Compute the unit cost and total income under:
(a) Absorption costing
(b) Marginal costing
iii) Reconcile the difference between the total income under absorption and marginal costing.

b) A company is engaged in manufacturing two products A and B. Product A uses one unit of component X and two units
of component Y. product B uses two units of component X and one unit of component Y and two units of component Z.
component Z which is assembled in the factory uses one unit of component Y.
Components X and Y are purchased from the market. The company has prepared the following forecast of sales and
inventory for the next year:
Product A B
(units) (units)
Sales 80,000 1,50,000
Stock at the end of the year 10,000 20,000
Stock at the beginning of the year 30,000 50,000

The production of both the products and the assembling of the component Z will be spread out uniformly
throughout the year. The company at present orders its inventory of X and Y in quantities equivalent to 3 months
production. The company has complied the following data related with the two components:
X Y
Price per unit (Rs.) 20 8
Order placing cost per order (Rs.) 1,500 1,500
Carrying cost per annum 20% 20%

Prepare production budget, component budget and calculate EOQ of components X & Y. 8

Answer:
a)
i) Under or over absorption of overhead: Rs.
Budgeted Fixed Overhead 72,000 Hrs. × Rs.6 4,32,000
Less: Actual Overhead less than Budgeted Fixed Overhead __ 48,000
Actual Fixed Overhead 3,84,000
Budgeted Variable Overhead (72,000 Hrs. × Rs.4) 2,88,000
Add: Actual Overhead higher than Budgeted 20,000
Actual Variable Overhead 3,08,000

Fixed & Variable Overhead applied 72,000 Hrs × Rs.10 7,20,000


Actual Overhead (3,84,000 + 3,08,000) 6,92,000
Over Absorption __28,000

ii) Computation of Unit Cost & Total Income


Unit Cost Absorption Costing Marginal Costing
(Rs.) (Rs.)
Direct Material 16.00 16.00
Direct Labour 54.00 54.00
Variable Overhead 12.00 12.00
Fixed Overhead 18.00 ____-
Unit Cost 100.00 82.00
Income Statements
Absorption Costing
Sales (21500 × Rs.168) 36,12,000
Less: Cost of goods sold (21500 × 100) 21,50,000
Less: Over Absorption ___28,000 __21,22,000
14,90,000
Less: Selling & Distribution Expenses (2,500×20=760,000) 11,90,000
Profit 3,00,000
(35)

Marginal Costing
Sales 36,12,000
Less: Cost of goods sold (21500×82) 17,63,000
Add: Under Absorption 20,000 17,83,000
18,29,000
Less: Selling & Distribution Expenses (Variable only) 4,30,000
Contribution 13,99,000
Less: Fixed Factory and Selling & Distribution
Overhead (3,84,000 + 7,60,000) 11,44,000
Profit 2,55,000

(iii) Reconciliation of Profit


Difference in Profit: = Rs.3,00,000 – 2,55,000
= Rs.45,000

Due to Fixed Factory Overhead being included in Closing Stock in Absorption Costing but not in Marginal Costing such
difference arises. Therefore, difference in Profit:
= Fixed Overhead Rate (Production – Sale)
= 18× (24,000 – 21,500)
= 45,000

b) Production Budget
Product “A” Product “B”
Sales 80,000 1,50,000
Add: Closing stock 10,000 20,000
Less: Opening stock 30,000 50,000
Production Budget 60,000 1,20,000
Budget of Component
Component X Y Z
Product A: Production 60,000 units 60,000 1,20,000
Product B: Production 1,20,000 units 2,40,000 1,20,000 2,40,000
Component Z: 2,40,000 units 2,40,000
Total 3,00,000 4,80,000 2,40,000

Optimal order quantity of components X & Y


Component X Y
Order placing costs Rs. 1,500 1,500
Price of the component Rs. 20 8
Carrying cost @ 20% Rs. 4 1.60

X Y
(2  3,00,000  1,500) (2  4,80,000 1,500)
EOQ =
4 1.60
= 15,000 = 30,000

4)
a) At the end of first year on 31st March, 2011 in the books of ABC Constructions Ltd. the Bridge Contract Account stands
debited with the cost of material issued, labour, overheads expended and plant issued and it stands credited with material at
site Rs. 25,000; material returned Rs. 15,000 and plant at site Rs. 476,000 after charging depreciation at 15 percent. The
material issued, labour, overheads and plant issued debited to the contract account, are in the ratio of 5: 4: 2: 4. The
contractee's architect had certified 75 percent of the contract as completed at the end of the year and 90 percent of the
certified work value had been received in cash Rs. 1,620,000. The accounts department informs that 2/3 of the profit on cash
basis credited to Profit and Loss account on the contract is Rs. 213,600

You are required to prepare the Bridge Contract Account showing the cost of work done but uncertified. 8

b) The existing incentive system of Beta Ltd. is as under:


Normal working week : 5 days of 8 hours each plus 3 late shifts of
3 hours each.
Rate of payment : Day work: Rs. 160 per hour
(36)

: Late shift: Rs. 225 per hour


Average output per operator for 49 hours
week i.e. including 3 late shifts : 120 articles.

In order to increase output and eliminate overtime, it was decided to switch on to a system of payment by results. The
following information is obtained:
Time-rate (as usual) : Rs. 160 per hour
Basic time allowed for 15 articles : 5 hours
Piece-work rate : Add 20% to basic piece-rate
Premium Bonus : Add 50% to time.

Required:
Prepare a statement showing hours worked, weekly earnings, number of articles produced and labour cost per article for one
operator under the following systems:
i) Existing time-rate
ii) Straight piece-work
iii) Rowan system
iv) Halsey premium system
Assume that 135 articles are produced in a 40 hour week under straight piece work, Rowan premium system, and Halsey
premium system above and worker earns half the time saved under Halsey premium system. 8

c) X Ltd. is reviewing its stock policy, and has the following alternatives available for the
evaluation of stock:
i) Purchase stock twice in a month, 400 units.
ii) Purchase monthly, 800 units
iii) Purchase every three months, 2,400 units
iv) Purchase every six month, 4,800 units
v) Purchase annually, 9,600 units
It is ascertained that the purchase price per unit is Rs. 40 for deliveries up to 2,000 units. A 5% discount is offered by the
supplier on the whole order where deliveries are 2,001 to 4,000 units and 10% reduction on the total order for deliveries in
excess of 4,000 units. Each purchase order incurs administration costs of Rs. 250. Interest on capital and other storage costs
are Rs. 12.50 per unit of average stock quantity held.
Calculate the optimum order size. 4

Answer:
a) Bridge Contract Account for the year ended 31 st March, 2011
Particulars Rs. Particulars Rs.
To Material issued 700,000 By Material at site 25,000
To Labour 560,000 By Material returned 15,000
To Overheads 280,000 By Plant at site 476,000
To Plant issued 560,000 By Work-in-Progress A/c:
To Notional Profit [W.N. 2] 356,000 Work certified 1,800,000
Work uncertified 140,000* 1,940,000
2,456,000 2,456,000
To P/L A/c 213,600 By Notional Profit b/d 356,000
To Reserve A/c 142,400
356,000 356,000
* Balancing figure.

Working Notes:
1. Calculation of the amount of Material issued, Labour, Overheads and Plant issued:
(i) Cost of Plant issued = Rs. 476,000 × 100/85 = Rs. 560,000. [Depreciation @ 15%]

(ii) The ratio of Material issued, Labour, Overheads and Plant issued had been given 5 : 4 : 2 :
4. So, the element wise amount would be:
Material issued Rs. 560,000 × 5/4 = Rs. 700,000
Labour Rs. 560,000 × 4/4 = Rs. 560,000
Overheads Rs. 560,000 × 2/4 = Rs. 280,000
2. Calculation of amount of Notional Profit:
Profit taken to P/L A/c = Notional Profit × 2/3 × Cash Received/Work Certified.
Rs. 213,600 = Notional Profit × 2/3 × 90/100.
Notional Profit = Rs. 213,600 × 3/2 × 100/90
= Rs. 356,000.
(37)

3. Value of Work certified: Cash received being 90% of Work certified.


Cash Received × 100/90
= Rs. 1,620,000 × 100/90 = Rs. 1,800,000

b) Table showing Labour cost per article:

Method of payment Hours Weekly Number of Labour cost


Worked earnings articles per article
Rs. produced Rs.

Existing time rate 49 8,425.00 120 70.21


Straight piece rate system 40 8,640.00 135 64.00
Rowan Premium system 40 9,007.41 135 66.72
Halsey Premium system 40 8,600.00 135 63.70
Working notes:

Existing time rate Rs.


Weekly wages 40 hours @ Rs. 160 per hour 6,400
9 hours @ Rs. 225 per hour __2,025
8,425
Piece Rate system:
Basic time: 5 hours for 15 articles (120/40hour=3hr/Article)
Cost of 15 articles at hourly rate of Rs. 160/hr. 800
Add 20% __160
960

Rate per article = Rs. 960/15 = Rs. 64


Earning for the week = 135 articles ×Rs. 64 = Rs. 8,640

Rowan Premium system:


Basic Time : 5 hours for 15 articles
Add : 50% to time
7.5 hours for 15 articles
Or 30 minutes per article
Time allowed for 135 articles = 67.5 hours
Actual time taken for 135 articles = 40 hours

 TA  HW 
Earnings = (HW ×RH) +   HW  RH 
 TA 
 67.5  40 
= (40 hrs. ×Rs. 160) +   40  Rs .160 
 67.5 
= Rs. 9,007.41

Halsey Premium System:


50
Earnings = HW ×RH + (TA –HW) ×RH
100
= 40× Rs. 160+ ½ (67.5 -40) ×Rs. 160
= Rs. 8,600

c) The purchase cost is not constant per unit. It is therefore, not possible to use the EOQ formula. For optimum order
size statement of cost is prepared as follows:
Annual Purchase Cost Storage Admin Total Cost
Order size No. of Order
Rs. Rate/unit Cost (Rs) Cost (Rs) (Rs)
9,600 1 345,600 36 60,000 250 405,850
4,800 2 345,600 36 30,000 500 376,100
2,400 4 364,800 38 15,000 1,000 380,800
800 12 384,000 40 5,000 3,000 392,000
400 24 384,000 40 2,500 6,000 392,500
The optimum order size is 4,800 units which cost Rs. 376,100 in total.
(38)

Working Notes:
(i) Order size up to 2,000 units, Purchase price is Rs. 40
Order size 2,001 – 4,000 units, Purchase price is Rs. 40 – 5% Discount = Rs. 38.
Order size greater than 4,000 units, Purchase price is Rs. 40 – 10% Discount = Rs. 36.
(ii) Storage cost: Rs. 12.50 per unit of average stock
For order size 400 units, Average stock 200 units × Rs. 12.50 = Rs. 2,500 and so on.

5) Distinguish between: (4×2.5 =10)


a) Opportunity cost and Imputed cost
b) Cost control and Cost reduction
c) Waste and Scrap.
d) Job costing and Process costing

Answer
a) Opportunity cost refers to the value of sacrifice made or benefit of opportunity forgone in accepting an alternative
course of action. For example, a firm financing its expansion plan by withdrawing money from its bank deposits. In such a
case the loss of interest on the bank deposit is the opportunity cost for carrying out the expansion plan. It is the cost of any
activity measured in terms of the best alternative forgone. It is the sacrifice related to the second best choice available to
someone who has picked among several mutually exclusive choices. The notion of opportunity cost plays a crucial part in
ensuring that scarce resources are used efficiently. Thus opportunity costs are not restricted to monetary or financial costs:
the real cost of output forgone, lost time, pleasure or any other benefit that provides utility should also be considered
opportunity costs.
Imputed cost is a cost that is incurred by virtue of using an asset instead of investing it or undertaking an alternative
course of action. An imputed cost is an invisible cost that is not incurred directly, as opposed to an explicit cost, which is
incurred directly. It is also known as notional cost. Imputed cost is a hypothetical cost and it does not appear in financial
records. But it is relevant for decision making. Interest on capital is a common type of imputed cost. Financial accountancy
accords recognition to interest on capital only if it is actually paid or constitutes a legal liability. If desirability of a project is
being evaluated, failure to consider interest cost may result in an erroneous decision. For all practical purpose there is no
difference in opportunity costs and imputed costs.

b) Cost Control and Cost Reduction: Cost control is operated through setting standards of targets and comparing actual
performance therewith, with a view to identify deviations from standards or norms and taking corrective action in order to
ensure that future performance conforms to standards or norms. Cost reduction is a continuous process of critical cost
examination, analysis and challenge of standards. Each aspect of business viz., products, process, procedures, methods,
organization, personnel, etc. is critically examined and reviewed with a view of improving efficiency and effectiveness and
reducing the costs. Cost control is a method whereby costs are managed and monitored to ensure they do not grow beyond
the projected amount.
Cost control lacks the dynamic approach which planned cost reduction demands. In cost reduction, standards which are the
basis of control are constantly challenged for improvement. Cost reduction is a method whereby costs are reduced from the
projected amount.

c) Waste and Scrap


Waste represents the portion of basic raw materials lost in production process having no recoverable value. Waste may be
visible – remnants of basic raw materials or invisible e.g. disappearance of basic raw materials through evaporation, smoke
etc. Shrinkage of material due to natural causes may also be a form of a material wastage. In the production process, waste
of material does occur, such waste has no recoverable scheme, hence there is no chance of recovery from it.
Scrap is the incidental material residue coming out of certain types of manufacturing processes, usually of small amount and
low value, recoverable without further processing. Scrap is visible and do have recoverable value. Material defective goods
may be sold as scrap.

d) Job costing relates to a costing system where each unit or batch of output of products or services is unique. This creates
the need for the cost of each unit or batch to be calculated separately. Direct costs and factory overheads are allocated to
individual units of production and the finished goods stock consists of stock of unlike units. In contrast, a process costing
system relates to the situation where masses of identical units or batches are produced thus making is unnecessary to assign
costs to individual units or batches of output. Instead, the average cost per unit or batch of output is calculated by dividing
the total costs assigned to a product or services of the period by the number of units or batches of output for that period.
Direct costs and factory overhead costs are allocated to processes. When units are completed, they are transferred to finished
goods stock at average unit cost. Therefore, the finished goods stock consists of stock of like units valued at average unit
cost of production.
(39)

The main difference between the two is the cost object used for cost accumulation. Job costing is when for example a
tradesman comes to give you a quote for how much he is willing to do the job/repair that you want to be done, whereas
process costing are what a business has to spend in order to keep functioning, overheads etc.

6) Write short notes on: (4×2.5 =10)


a) Functions of management accounting system
b) Limitations of uniform costing
c) Information required for preparation of cost audit
d) Volume-based and non-volume-based cost drivers

Answer:
a) Management accounting systems provide information to assist managers in their planning and control activities.
Management accounting activities include collecting, classifying, processing, analyzing, and reporting information to
managers. It is so designed that its information help decision making within the firm. Its scope include information on sales
backlogs, unit quantities, prices, demands on capacity resources, and extensive performance measures based on physical or
non financial measures. The challenge is to develop management accounting practices that support the basic managerial
tasks of organizing, planning, and controlling operations to achieve excellence throughout the organization.

b) The following are the limitations of uniform costing:


(i) Sometimes it is not possible to adopt uniform standards, methods and procedures of costing in different firms due to
differing circumstances in which they operate. Hence, the adoption of uniform costing becomes difficult in such firms.
(ii) Disclosure of cost information and other data is an essential requirement of a uniform costing system. Many firms do
not wish to share such information with their competitors in the same industry.
(iii) Small firms in an industry believe that uniform costing system is only meant for big and medium size firms, because
they can't afford it.
(iv) It induces monopolistic trend in the business, due to which prices may be increased artificially and supplies withheld.

c) Preparation for cost audit of an organization requires the knowledge and gathering of following information and
data:
 Cost accounting system used in the organization,
 Production methods and manufacturing processes,
 Raw materials and components used in production,
 Cost records and documents,
 Cost accounting rules or cost accounting manual used in the organization,
 Important information mentioned about the costing requirement in Memorandum and Articles of Association,
etc.

d) Cost drivers are the factors, forces or events that influence and determine the cost of a particular activity and help
in assigning the cost to production unit. Direct costs are traced to cost and they themselves are cost drivers. When indirect
costs are assigned or allocated to product or services the factors, such as number of production run, number of machine set
up, number of purchase order etc. are used which are known as cost drivers. Cost drivers are unique physical aspects of the
production process which can be viewed as causing the cost pools to be incurred.

Volume-based cost drivers assume that a product‟s consumption of overhead resources is directly related or highly
correlated to production volume or unit. Some examples of volume-based cost drivers are units of output, direct labour
hours, machine hours, etc. There are some activities performed which are not directly related to volume of production such
as production run, machine set up, purchase order etc. The cost of these activities are allocated to the product on the basis of
non-volume-based cost drivers, such as number of production run, number of machine set up, number of purchase order etc.
Both costs derivers are meant for cost control. Volume-based cost drivers facilitate to control cost directly in proportion to
production output. Non volume-based cost drivers facilitate to control cost through parameters not directly linked to
production volume.
The Institute of Chartered Accountants of Nepal
Suggested Answers of Financial Management
CAP II Examination – June 2011

1. A plastic manufacturer has under consideration the proposal of production of high quality plastic glasses. The necessary
equipment to manufacture the glasses would cost Rs. 80,000. Investment allowance rate on purchases of equipment is
20%. The production equipment would last 5 years with no salvage value. The glasses can be sold at Rs. 3 each.
Regardless of the level of production, the manufacturer will incur cash costs of Rs. 25,000 each year, if the project is
undertaken. The overhead costs allocated to this new line would be Rs. 5,000. The variable cost is estimated at Rs. 2.0
per glass. The manufacturer estimates it will sell about 75,000 glasses per year; the straight line method of depreciation
will be used; the applicable tax rate is 55%.
a) Calculate the cash outflows of the project. 5
b) Determine the project‟s total present value at 0, 10, 20, 30 and 40 percent discount rate. 5
c) Present the net present value profile for the proposal. 3
d) Explore the relationship between Pay Back Reciprocal and IRR? 5
e) What is the basic assumption behind terminal Value Approach? 2

You can take the help of following PV table:


Year 10% 20% 30% 40%
1 0.909 0.833 0.769 0.714
2 0.826 0.694 0.592 0.510
3 0.751 0.579 0.455 0.364
4 0.683 0.482 0.350 0.260
5 0.621 0.402 0.269 0.186

Answer No. 1
(i) Cash outflows:
Particulars Rs
Cost of new equipment purchases 80,000
Less: Investment Tax Credit (Rs16,000) x 55% 8,800
Net cash outflow 71,200
Cash inflows:
Particulars Rs
Sales Revenue 225,000
Less Costs:
Variable Costs 150,000
Additional Fixed Cost 25,000
Additional Depreciation 16,000
Earning Before Taxes 34,000
Less Taxes 18,700
Earning After Taxes 15,300
Add Depreciation 16,000
Cash Flow After Tax (t = 1 – 5) 31,300
(Note: Costs allocated from other departments will not be considered as they do not involve any corresponding
incremental cash outflows)
(ii) PV at different rates of discount:
Rate of discount (%) PV factor Time (Years) CFAT (Rs.) Total PV (Rs.)
0 5.000 1-5 31,300 156,500
10 3.791 1-5 31,300 118,658
20 2.991 1-5 31,300 93,618
30 2.436 1-5 31,300 76,247
40 2.035 1-5 31,300 63,696
(iii)
Net present value profile for the project:
Rate of discount (%) NPV (Rs.)
0 85,300
10 47,458
20 22,418
30 5,046
40 (7,505)

NMG P.T.O.
(41)

(iv) The reciprocal of the pay back is a good approximation of the IRR. The pay back period reciprocal can be applied to
both annuity and mixed streams of cash flows. In case of annuity, pay back period of the proposed investment project is
determined and factor closest to the pay back period in the year row is looked into. In case of mixed stream cash flows,
average annual cash inflow is first calculated and approximated IRR is determined with the help of „fake‟ pay back period.
(v) Basic assumption behind the Terminal Value approach is that each cash inflow is re-invested in another asset at a certain
rate of return from the moment it is received until the termination of the project.
2. Following book value capital structure is available in respect of PQR Ltd.
(Rs. in million)
__________________________________________________________________________________
_______
Equity Capital (in shares of Rs. 100 each, fully paid-up at par) 150
11% Preference Capital (in shares of Rs. 100 each, fully paid-up at par) 10
Retained Earnings 200
13.5% Debentures (of Rs. 100 each) 100
15% Term Loan 125
__________________________________________________________________________________
_______
The next expected dividend per share on equity shares is Rs. 36 and the dividend per share is expected to grow at the rate of
7%. The market price per share is Rs. 400.
Preference stock, redeemable after 10 years, is currently selling at Rs. 75 per share. Debentures, redeemable after 6 years,
are selling at Rs. 80 per debenture. The income tax rate for the company is 25%.

You are required to: (8 +7=15)


a) Calculate the weighted average cost of capital using market value proportion., and
b) Determine the weighted marginal cost of capital for the company, if it raises Rs. 100 million next year, given the
following information:
 The amount will be raised by equity and debt in equal proportions.
 the company expects to retain Rs. 15 million earnings next year.
 the additional issue of equity shares will result in the net price per share being fixed at Rs. 320.
 the debt capital raised by way of term loan will cost 15% for the first Rs. 25 million and 16% for the next Rs. 25
million.

Answer No. 2
Working Notes:
(1) Cost of Equity Capital (Ke) and Cost of Retained Earnings (Kr)
Ke = D1/P0 + g = 36/400 + 0.07 = 0.09 + 0.07 =0.16 or 16%

(2) Cost of Preference Share Capital (Kp)


Kp = D + (Rv – Sv) / N = 11 + (100 – 75) / 10 = 11 + 2.5 = 0.1543 or 15.43%
(Rv + Sv) /2 (100 + 75) / 2 87.5
(3) Cost of Debentures (Kd)
Kd = I + (Rv – Sv) / N (1 – t) = 13.5 + (100 – 80) / 6 (1 – 0.25) = (13.5 + 3.33) 0.75
(Rv + Sv) /2 (100 + 80) / 2 90
= 0.14025, say, or 14.03%

Alternatively
Cost of Debenture (Kd)
Kd = I(1 - t) + (Rv – Sv) / N =
(Rv + Sv) /2
Kd =13.5 (1 –0.25) +(100-80)/6
(100 + 80) /2
= 10.125+3.333
90
= 13.458
90
= 0.1495 or 14.95%
Note: Students may also use either of one formula to calculate the cost of debenture and same can be used to calculate
WACC. In the solution first alternative is used.

(4) Cost of Term Loan (Kt)


Kt = I (1 – t)
0.15 (1 – 0.25) = 0.15 x 0.75 = 0.1125 or 11.25%
On first Rs. 25 million Term Loan = 0.15 (1 – 0.25) = 0.1125 or 11.25%
(42)

On the next Rs. 25 million Term Loan = 0.16 (1 – 0.25) = 0.12 or 12%
(5) Cost of Fresh Equity Shares (Ke)
Ke = D1/P0 + g = 36/320 + 0.07 = 0.1825 or 18.25
(i) Calculation of Weighted Average Cost of Capital (WACC) using market value proportion:
____________________________________________________________________________________________
____
Source of Finance Market Value Weight Cost of Weighted cost
(Rs. Millions) Capital of Capital %
____________________________________________________________________________________________
_______
Equity Capital 600.00 0.739 0.1600 0.11824
(1.5 million shares x Rs. 400
11% Preference Capital
(1 lakh shares x Rs. 75) 7.50 0.009 0.1543 0,00139
13.5% Debentures 80.00 0.098 0.1403 0.01375
(1 million debentures x Rs, 80)
15% Term Loan 125.00 0.154 0.1125 0.01733
812.50 WACC: 0.15071
____________________________________________________________________________________________
_______
Therefore, WACC = 15.07%
Note: Retained earnings are not considered for calculating WACC since it does not have any market value
separately. The market value of equity shares reflects the value of retained earnings as well.
* Alternatively 0.1495 or 14.95% can be used to calculate WACC.

(ii) Calculation of WACC of PQR Ltd. when it raises Rs. 100 million next year:
____________________________________________________________________________________________
____
Source of Finance Amount Weight Cost of Weighted cost
(Rs. Millions) of Capital of Capital %
____________________________________________________________________________________________
_______
Retained Earnings 15 0.15 0.1600 0.02400
Debt 15 0.15 0.1125 0.01688
Equity Shares 10 0.10 0.1825 0.01825
Debt 10 0.10 0.1125 0.01125
Equity Shares 25 0.25 0.1825 0.04563
Debt 25 0.25 0.1200 0.03000
100 0.14601
Therefore, WACC of raising Rs.100 million next year = 14.60%

3.
a) Progressive Manufacturers Ltd. has sales of Rs. 250 million of which 80 per cent is on credit basis. The present credit
terms of the company are “2/15, net 45”. At present, the average collection period is 30 days. The proportion of sales on
which customers currently take discount is 0.50.
The firm is considering relaxing its discount terms to “3/15, net 45”. Such a relaxation is expected to increase current
credit sales by Rs. 10 million, reduce the average collection period to 27 days and increase the proportion of discount
sales to 0.60. The average selling price of he company‟s product is Rs. 1,000 per unit and variable cost per unit works
out to be Rs. 800. The company is subject to a tax rate of 25 per cent and it‟s, before tax rate of borrowings for working
capital is 12 per cent.
Should the firm change its credit terms to “3/15, net 45”? Support your answers by calculating the expected change in
net profit (assume 360 days in a year) 8
(* Note: In the question the information is found missing "by Rs. 10 million, reduce the average collection period to 27
days and increase the proportion of discount sales to 0.60.")

b) Describe the term “beta co-efficient” as used in the portfolio theory. Explain what does the value of beta of 1, less than
1 and more than 1 signify. (5+2=7)

Answer No. 3
a)
Total Sales = Rs. 250 million
Credit Sales = Rs. 250 x 0.80 = Rs. 200 million
(a) Present Credit Policy:
(43)

Present credit terms are „2/15, net 45‟


Present average collection period = 30 days
Proportion of sales on which customers currently take discount is 0.5, or 50%.
(b) Basic revenue and cost structure applicable to both the policies:
Selling price per unit: Rs. 1,000
Variable cost per unit: Rs. 800
Contribution per unit: Rs. 200
P / V Ratio = 200/1,000 x 100 = 20%
Contribution from increased sales = Rs. 1,000,000 x 20% = Rs. 200,000
(c) Relaxed Credit Policy:
Reduction in average collection period to 27 days
Increase in proportion of discount sales to 0.60, or 60%
Change in the investment of Receivables
= [Rs. 200 million x 27 – 30 ] + [Rs. 10 million x 80/100 x 27/360]
360
= (-) Rs. 1.667 million + Rs. 0.60 = (-) Rs. 1.0667 million
Therefore, the reduction in receivables investment is Rs. 1.0667 million.
Saving in cost from reduction in receivables investment
= 1.0667 x 0.12 (1 – 0.25) = 0.096 million
Increase in discount cost
= [(Rs. 200 + 10) x 60/100 x 3/100] – (200 x 50/100 x 2/100)
= 3.78 – 2 = Rs. 1.78 million
Statement showing Profitability of Relaxing Credit Policy
(Rs. in million)
Contribution from Increased Sales 2.000
Cost savings from Reduction in the Receivables Investment 0.096
2.096
Less: Incremental Discount Cost 1.780
Incremental Profit 0.316
The firm can increase its profits by Rs. 0.316 million by relaxing the credit policy. Therefore, it is suggested to change the
credit terms to „3/15, net 45‟ from the present “2/15, net 45”.

b) Under capital asset pricing model (CAPM), the risk of an individual security can be estimated. The market related risk,
which is also called „systematic risk‟ is unavoidable even by diversification of the portfolio. The systematic risk of an
individual security is measured in terms of its sensitivity to market movements which is referred to as security‟s beta.
Beta coefficient is a measure of the volatility of stock price in relation to movement in stock index of the market. Thus, beta
is the index of systematic risk. The beta factor of the market as a whole is 1.0. A beta of 1.0 of individual security indicates
the average level of risk as compared to the market.
Mathematically, the beta coefficient of a security is the security‟s covariance with the market portfolio divided by the
variance of the market portfolio. Symbolically,
βi = Cov im. = σi σm Cor im, where
Varm σm2
βi = Beta of an individual security
Cov im. = Covariance of returns of individual security with the market portfolio
Varm = Variance of returns of market portfolio (σ m2)
Cor im = Correlation coefficient between the return s of individual security and the market portfolio
σi = Standard deviation of returns of individual security
σm = Standard deviation of returns of market portfolio
The degree of volatility can be expressed as follows:
 If beta is 1, then it has the same level of risk profile as the market as a whole.
 If the beta is less than 1, it is not as sensitive to systematic or market risk as the average investment.
 If beta is more than 1, it is more sensitive to the market risk than the average investment.

4.
a) Based on the credit rating of the bonds, an investor has decided to apply the following discount rate for valuing the
bonds.
Credit rating Discount rate
AAA 364-day Treasury-bill rate + 3% spread
AA AAA + 2% spread
A AAA + 3% spread
(44)

The investor is considering investing in an AA rated, Rs. 1,000 face value bond currently selling at Rs. 1,010. The bond has
five years to maturity and the coupon rate on the bond is 15% per annum payable annually. The next interest payment is due
one year from today and the bond is redeemable at par. (Assume 364-day Treasury bill rate to be 9%)
You are required to calculate: (5+3=8)
i) Intrinsic value of the bond for the investor. Should the investor invest in the bond?
ii) Current yield (CY) and the yield to maturity (YTM) of the bond.

b) The capital structure of Stable Ltd. is extracted below:


(Rs. in Million)
Equity capital: 100 thousand shares of Rs.100 each 10.0
Reserve and surplus 12.0
12% preference shares: 55,000 shares of Rs. 100 each fully paid up 5.5
14% debentures of Rs. 1,000 each; 3,000 numbers 3.0
Long-term loan from financial institution at 12% per annum 2.0
32.5
The company is also availing a bank overdraft of Rs. 2 million carrying interest at 15% per annum. The company is now
drawing up its profit plan for the next year. It wants to pay dividend to equity shareholders at 15% and keep the total
dividend payout (equity as well as preference shareholders) at 60%.
Assuming that the tax rate applicable to the company is 25%., what level of earnings (EBIT) should the company try to
achieve to meet its plan?
7
Answer No. 4
a) AA rated face value of bond = Rs. 1,000
Current selling price = Rs 1,010
Maturity period of bond = 5 years
Coupon rate of the bond = 15% per annum payable annually
Bond redeemable at par at the end of 5th year.
Net interest payment is due on year from today.
Discount rate for AA rated bond = 9% + 3% + 2% = 14%
Calculation of Present Value of Cash Inflow from Bond
(Rs.
___________________________________________________________________________________________________
_________________________________________________________
Year-end Cash Inflow PV Factor at 14% Present Values
___________________________________________________________________________________________________
________
1 150 0.8772 131.58
2 150 0.7695 115.43
3 150 0.6750 101.25
4 150 0.5921 88.82
5 1,150 0.5194 597.31
Present value of total Cash Inflow: 1,034.40

Thus, the intrinsic value of bond is Rs. 1,034.40. Since the intrinsic value of bond (Rs. 1,034.40) is more than its current
market value (Rs. 1,010), it is suggested to purchase the bond.

Current yield = Annual Bond Interest x 100 = 150 x 100 = 14.85%


Market price 1,010
Yield to Maturity (YTM)
P = Rs. 150 x PVIFA @ 15% for 4 years + Rs. 1,150 x PVIF at 15% for 5 th year
= (150 x 2.855) + (1,150 x 0.4972 = 428 + 571.78 = 1,000.03
Present value at 14% = Rs. 1,034.40
Present value at 15% = 1,000.03
By interpolation, YTM = 14 + 1,034.40 – 1,010 x 1 = 14% + (24.40 / 34.37) = 14.71%
1,034.40 – 1,000.03
b)
Let „x‟ be the EBIT to meet the company‟s commitments.
Interest Payable Yearly (Rs. in Million)
On debentures @ 14% on Rs. 3 million 0.42
On long term loan of Rs. 2 million @ 12% 0.24
On bank overdraft of Rs. 2 million @ 15% 0.30
0.96
Profit before tax (PBT) = EBIT – 0.96 = x – 0.96
(45)

Tax at 25% = (x – 0.96) / 4


Profit after tax (PAT) = 3 (x – 0.96) / 4

Total dividend payable (Rs. in Million)


On preference capital of Rs. 5.5 million @ 12% 0.66
On equity capital of Rs. 10 million 1.50
2.16

Total dividend payout is limited to 60% of PAT and is also equal to Rs. 2.16.
Therefore, 3 (x – 0.0.96) /4 x 60 / 100 = 2.16
Or 3(X-0.96)/4=216/60
Or X-0.96=3.6x4/3
Or, x – 0.96 = 4.80
Or, x = 4.80 + 0.96 = 5.76
Hence, earnings before interest and tax should be Rs. 5.76 million.

5.
a) The following information pertains to a company:
Net profit (Rs. in „000) 60,000
12% preference shares capital (Rs. in „000) 20,000
Number of equity shares outstanding (in „000) 60
Return on investment 20%
Equity capitalization rate 16%
You are required to: (4.5+1.5=6)
i) Compute the dividend payout ratio so as to keep the share price at Rs. 412.50 by using Walter model, and
ii) Ascertain (giving reasons) the optimum payout ratio if return on investment is 16% and equity
capitalization rate is 18%.
a) The beta co-efficient of security X is 1.6. The risk free rate of return is 12% and the required rate of return is 18% on
the market portfolio. If the dividend expected during the coming year is Rs. 25 and the growth rate of dividend and
earnings is 8%, at what price should the security X can be sold based on the capital asset pricing model. 5
b) An investor saw an opportunity to invest in a new security with excellent growth potential. He wants to invest more
than he had, which was only Rs. 100,000. He sold another security short with an expected rate of return of 15%. The
total amount he sold was Rs. 400,000, and the total amount he invested in the growth security, which had an expected
rate of return of 30%, was Rs. 500,000. Assuming no margin requirements, what is the investor‟s expected rate of
return? 4

Answer No. 5
a) Calculation of Earnings per Share (EPS) (Rs. in „000)
Net Profit 60,000
Less: Preference Dividend (20,000 X 12 / 100) 2,400
Net Profit after Preference Dividend 57,600
Earnings per Share in Rs. [Rs 57,600,000/60,000] 960

(i) Calculation of Dividend Payout Ratio:


Let dividend payout ratio be „x‟. The formula for share price under Walter model is::
P = D + r / Ke (E – D), where
Ke
P = Market price per share (Rs. 412.50 Given)
E = Earnings per share (Rs. 960 derived above)
D = Dividend per share (Rs. 960 x Given)
r = return on investment (0.20 given)
Ke = Cost of equity (0.16 Given)
Substituting the values, we get:
412.50 = 960 x + 0.20/0.16 (960 – 960 x)
0.16
Or, 412.50 = 60 x + 1.25 (60 – 60 x)
0.16
Or, 412.50 = 60 x + 75 - 75 x
0.16
Or, 66 = 1,200 – 240x
Or, x = 4.725, or 472.5%
Thus, the required dividend payout ratio is 472.5%.
(ii) Optimum Payout Ratio when Return on Investment (16%) is less than Equity Capitalization Rate (18%)
(46)

According to Walter model, when return on investment is less than the cost of capital, the value of the share is highest when
dividend payout is maximum. It is evident that when r/Ke is less than 1, higher dividend will maximize the value per share.
Therefore, the dividend payout should be 100% in this case.
b) Expected rate of return is calculated as follows by applying CAPM formula:
E (Ri) = Rf + Bi (Rm – Rf)
= 12% + 1.6 (18% - 12%) = 12% + 9.6% = 21.6%.
Price of security X is calculated with the use of dividend growth model formula as follows:
Re = D1 / P0 + g, where
D1 = Expected dividend during the coming year
Re = Expected rate of return on security X
g = Growth rate of dividend
P0 = Price of security X
Substituting the values, we get:
0.216 = 25/ P0 + 0.08,
Or, 0.216 = 2.50 + 0.08 P0
P0
Or, 0.216 P0 = 25 + 0.08 P0
Or, 0.216 P0 – 0.08 P0 = 25,
Or, 0.136 P0 = 25
Or, P0 = 25 / 0.136 = Rs. 183.82.
The price at which the security X should be sold as per CAPM is Rs. 183.82.
c)
Computing the portfolio weights for each security is done with the formula:
Investment in A (sold short)
Total equity investment
From the given problem, we find:
WA = - Rs, 400,000/ Rs. 100,000 = - 4.0
WB = Rs. 500,000 / Rs. 100,000 = 5.0
Rp = (- 4 x 0.15) + (5 x 0.30) = - 0.60 + 1.50 = 0.90, or 90%.
Thus, the expected rate of return on this portfolio is 90%.

6. Write short notes on: (4×2.5=10)


a) Factoring services
b) Commercial paper
c) Perpetuities
d) Inflation Premium

Answer No. 6
a) Factoring services - Factoring is a unique financial innovation. It is both a financial as well as a management
support to a client. It is a method of converting a non productive, inactive asset like receivable into a productive asset like
cash by selling receivables to a company that specializes in their collection and administration. Factoring is a business
involving a continuing legal relationship between a financial institution (factor) and a business concern (client) selling goods
or providing services to trade customers whereby the factor purchases the client's account receivable and in relation thereto,
controls the credit, extended to customers and administers the sales ledger. Basically three kinds of services fall into this:
sales ledger administration and credit management, credit collection and protection against default and bad debt losses,
financial accommodation against the assigned book debts.
b) Commercial Paper - It is an important money market instrument in advanced countries like USA to raise short term
funds. It is a form of unsecured promissory note issued by firms to raise short term funds. The commercial paper market in
the USA is a blue-chip market where financially sound and highest rated companies are able to issue commercial papers.
The buyers of commercial paper include banks, insurance companies, unit trusts and firms with surplus funds to invest for a
short period with minimum risk. Given this objective of the investors in the commercial paper market, there would be
demand for commercial papers of highly creditworthy companies.
c) Perpetuities can be defined as a stream of equal payments expected to continue for ever. Most annuities call for
payments to be made over some finite period of time, for example, Rs1000 per year for five years. However, some annuities
go for indefinitely, or perpetually, and these are called perpetuities. The present value of perpetuities is found as below:
PV (Perpetuities) = Payment/ Interest Rate
Most preferred stocks entitle their owners to regular, fixed dividend payments lasting forever. These are one of the examples
of „perpetuities‟.
d) „Inflation Premium‟ is the premium for expected inflation that investors add to the real risk free rate of return.
Inflation has a major impact on interest rates because it erodes the purchasing power and lowers the real rate of return on
investment. Investors are well aware of all this, so when they lend money, they build in an inflation premium equal to the
average inflation rate expected over the life of the security. Therefore, if the real risk free rate is 4 percent and if inflation is
expected to be 5% (and hence inflation premium=5%) during the next year, then the quoted rate of interest would be 9%.
(47)

7. Distinguish between: (4×2.5=10)


a) Asset Beta Vs. Equity Beta
b) Discrete Probability Distribution and Continuous Probability Distribution
c) Proxy fight and Takeover
d) Business Risk and Financial Risk
Answer No. 7
a) Assets of a leveraged firm are financed by debt and equity. Therefore, the assets beta should be the weighted
average of the equity beta and the debt beta. For an unlevered (all equity) firm, the asset beta and the equity beta would be
the same. Debt is less risky than equity. Hence the beta of debt will be lower than the equity beta. In case of the risk free
debt, beta will be zero. For a levered firm, the proportion of equity will be less than 1. Therefore, the beta of asset will be
less than the beta of equity.
There is also a linear relationship between the equity beta and the financial leverage. As the financial
leverage increases, the equity beta also increases. The equity beta is equal to the asset beta if debt is zero.
b) Discrete Probability Distribution and Continuous Probability Distribution
In case of „Discrete Probability Distribution‟ the number of possible outcome is limited or finite. Suppose, if we assume that
there will be only three states of economy; recession, normal or boom, this will be the example of discrete probability
distribution.
In other hand, if we assume that there will be unlimited or infinite number of possible outcomes that will be the
case of continuous probability distribution. With continuous distribution, it is more appropriate to ask what the probability is
of obtaining at least some specified rate of return than to ask what the probability is of exactly that rate.
c) Management always solicits stockholders‟ proxies and usually gets them. However if earnings are poor and
stockholders are dissatisfied, an outside group might solicit the proxies in an effort to overthrow management and take
control of the business. This is known as proxy fight.
Takeover is an action whereby a person or group succeeds in ousting a firm‟s management and taking control of
the company. In recent years there are cases, where attempts have been made by one corporation to take over another by
purchasing a majority of the outstanding stock.
d) Business Risk is defined as the uncertainty inherent in projections of future Return on Assets (ROA), or of Returns
on Equity (ROE) if the firm uses no debt. Business risk is the single most important determinant of capital structure.
Business risk varies from one industry to another and also among firms in given industry.
Financial risk is the additional risk placed on the common stockholders as a result of using financial leverage,
which results when a firm uses fixed income securities (debt and preferred stock) to raise capital. Thus, it is the portion of
stockholders‟ risk, over and above basic business risk, resulting from the manner in which the firm is financed.
The Institute of Chartered Accountants of Nepal
Suggested Answers of Business Communication and Marketing
CAP II Examination – June 2011

Section -'A' : Business Communication

1. Read the following case carefully and answer the questions given below:
The CEO of Giga Bank Ltd., with its head office based in Kathmandu has asked you, a Senior Recruitment Officer of the
bank to write a reply to Mrs. Lata Dawadi, a permanent and short-listed candidate of the bank, in response to her complaint
that a fresh candidate was selected ignoring her merit, work efficiency and experience. Reply in an encouraging and
convincing language including the following, 10
a) That the Selection Committee was aware of the complainant‟s qualifications, work efficiency and
experience,
b) That she could not be selected only for some technical reasons- the selected outside candidate has
long years of association with two or three banking organizations.
c) That the bank assures Mrs. Lata Dawadi that her appointment to a higher post will be considered in
case of any new opening in the future.
Answer No.1
Mrs. Lata Dawadi
Administrative Officer
Giga Bank Ltd.
Kathmandu

Regarding your complaint that an outside fresh candidate was selected to the post of the bank‟s senior administrative officer
and that you were left out as a disqualified candidate, we would like to persuade you that the bank management is well
aware of your qualifications, competence and work experience. But the interview board considered it necessary to pick up
the candidate on ground that the latter had the long years of association with two or three banking institutions. The
testimonials that one has submitted bear witness to the fact one had the wide experience in the related field. So, the selection
committee has made a wide decision to priotize the wider experience and higher qualification. So, we hope that you would
take it as a reasonable ground for making such a decision. Yet, we assure that the bank will take your experience and
qualification into account, in the further advertisement. In case of the new opening in future, bank assures to appoint you to
a higher post.

Wishing you a successful administrative career,

Senior Recruitment Officer

2. Analyze how information is transmitted through nonverbal messages and discuss how to improve nonverbal
communication skills. 10
Answer No.2
Nonverbal messages are sent by our eyes, face, and body. For example, sustained eye contact indicates
trust or admiration; brief eye contact may signify fear or stress. Expressions on a communicator‟s face
can supplement or entirely replace verbal messages. Posture can indicate status, confidence, shyness, or
submissiveness. Gestures also semd nonverbal messages, many of which are culture dependent.
Moreover, how a communicator uses time, space, and territory sends messages that require no words.
The amount of space we need for social interaction can be another means of sending messages
nonverbally.
Communicators may improve their nonverbal communication skills by maintaining eye contact, looking alert, eliminating
physical barriers that separate them from their listeners, and improving their comprehension of nonverbal signals. Besides,
following specific activities might prove valuable in improving nonverbal skill.
 Self-monitor your actions and tones. When you start to notice a habits or oddities that occur while
you speak, write them down. This will help correct yourself when it comes down to it.
 Be aware of how people see you. We all see ourselves one way, but others can see us in a different
manner. Notice how people react to you while you speak.
 Ask friends to help you determine where you're going wrong. They will know if you are sending
any conflicting signals. If you feel comfortable, ask your colleagues as well. They see another side
of you that your friends normally don't.

NMG P.T.O.
(49)

 Videotape yourself while you speak. Make sure you are able to see at least your upper half. Notice
any gestures, facial expressions, or tones that can conflict with the message you are giving with
your words.
 Adapt with the roles you have in your life. You act differently as a student and as a mother, for
example. Be sure that your words, tones, actions, and gestures correlate with the environment and
role that you are in at that time.

3. Self-dependent students, who are quite concerned with job-search, make a lot of efforts towards job seeking to
support their studies. During their job-search efforts, they perform various steps. Mention five major steps which most of
the students follow in their efforts towards job search. 10

Answer No.3
Self-dependent students, who do learning by earning, need to do a lot of job searching during their career development
period. Many of such students need to get some kind of job to support their education. As getting employment is not an easy
task, they will have to spend quite a considerable amount of time during job search. They do this by building a network of
contacts with a number of related personalities and organizational heads at many places. The job search follows a process
which takes the searcher to a wider circle of friends facing similar problems and those who have already gone a few steps
ahead of them in getting the job. The most probable people they meet include: university professors, business executives,
businessmen and industrialists. They also need to approach community organizations, NGOs and INGOs and make contacts
with them through internships The job search process involves some of the following steps

Identifying and Preparing for Appropriate jobs During job search; students have to consider both internal and external
factors to get the right job. During this time, greater efforts are made to get the job that matches with the applicant‟s
education, skills and experiences. Here, the job searcher‟s specific qualities, distinct personal qualifications, work
experience and special qualitiescommunication skills matter much for the potential employer. Most schools have career
counseling centers that provide career counseling on job during the study period of the student. Besides, classified
advertisements published in the local papers also help the students in getting appropriate jobs.

Preparing for Employment: The stage of job search is followed by the second step in which the student prepares
himself/herself in preparing application documents. Here, the student makes personal visits, does online contacts and faxes
or emails his/ her resume to various organizations. He may also write a persuasive letter for an interview for employment
with follow-ups.

After finding one, he or she writes an application to select a group of companies, decides on whether to send a message
alone with a resume or with a resume and a reference sheet.

Placing of Resume in Job Search: A job application followed by a resume knocks at the door of the employer, who sets the
applicant to a challenging task of an interview.

Understanding & Writing an Effective Resume: A resume (also called data sheet and curriculum vitae) is the formal
arrangement of a writer‟s personal inventory. It lists facts in some orderly way. Normally, one applies for jobs by using
resume and letters. Writing complete resume require meeting certain requirements: contents, presentations, format and style
with appropriate style.

Resume Writing Process: One‟s job search activities begin with writing a good resume. The success of a resume in getting a
job depends upon three things: (a) Planning, (b) Start writing and (c) Completing the writing.

Planning involves: i. analyzing the situation in which the purpose of your writing the resume should be clear, ii. gathering
information about what the work organization intends to seek in its new job hire, iii. selecting the right medium and iv.
organizing the information

The Writing Stage: Here, start writing your resume in properly worded sentence format. Write things in a professional tone
to capture the audience attention.

Completing the writing: This involves evaluating the contents, editing and rewriting and producing the message to the
concerned as per instructions of the employer

4. What importance does ethical communication have in business? Discuss some of the ways of maintaining ethical
standards in business. 10
(50)

Answer No.4
As a business communicator, you should understand basic ethical principles so that you can make
logical decisions when faced with dilemmas in the workplace. Professionals in any field must deal with
moral dilemmas on the job. However, just being a moral person and having sound personal ethics may
not be sufficient to handle the ethical issues that you may face in the workplace. On the job you will
face many dilemmas, and you will want to react ethically.
Taking ethics into consideration can be painful in the short term. But in the long term ethical behaviour makes sense and
pays off. Dealing honestly with colleagues and customers develops trust and builds stronger relationships. Many businesses
today recognize that ethical practices make good business sense. Ethical companies endure less litigation, less resentment,
and less government regulation. The following guidelines can help you set specific ethical goals and maintain a high ethical
standard.
Abiding by the Law. Know the laws in your field and follow them. Particularly important for business communicators are
issues of copyright law. Don't assume that Internet items are in the "public domain" and free to be used. Internet items are
also covered by copyright laws.
Telling the Truth. Ethical business communicators do not intentionally make statements that are untrue or deceptive. We
become aware of dishonesty in business when violators break laws, notably in advertising, packaging, and marketing. Half
truths, exaggerations, and deceptions constitute unethical communication. But conflicting loyalties in the workplace
sometimes blur the line between right and wrong.
Labeling Opinions. Sensitive communicators know the difference between facts and opinions. Facts are verifiable and often
are quantifiable; opinions are beliefs held with confidence but without substantiation. Stating opinions as if they were facts
is unethical.
Being Objective. Ethical business communicators recognize their own biases and strive to keep them from distorting a
message. Honest reporting means presenting the whole picture and relating all facts fairly.
Using Inclusive Language. Strive to use language that includes rather than excludes. Do not use expressions that
discriminate against individuals or groups on the basis of their gender, ethnicity, disability, or age. Language is
discriminatory when it stereotypes, insults, or excludes people.
Giving Credit. Ethical communicators give credit for ideas by (1) referring to originators names within the text, (2) using
quotation marks, and (3) documenting sources with endnotes, footnotes, or internal references. In school or on the job,
stealing ideas or words from others is unethical.

5. Briefly explain any four of the followings: (4×2.5=10)


a) Use of you-viewpoint language in business
b) Ethnocentrism
c) Good news letter
d) High-context vs. low-context cultures
e) Managing publicity communication

Answer No.5
a) In writing business messages, the use of the you-viewpoint language helps in the promotion of
goodwill of the organization among its clients and customers. As the you-viewpoint language
emphasizes the reader‟s interests and concerns rather than the writer‟s. This technique is very effective
to motivate, influence and win favor of the people. It emphasizes you and your and de-emphasizes we
and our. The you-viewpoint language is an attitude that places the reader in the center of things. It
involves being friendly and treating people the way they like to be treated. For instance, the statement
“I am happy to report…” places the writer at the center, whereas “you will be happy to know….”
places the reader at the center. The you-viewpoint language, therefore, provides the reader a kind of
psychological boost and confidence.

b) The belief in the superiority of one‟s own race is known as ethnocentrism, a natural attitude
inherent in all cultures. It is the view that uncritically presupposes that one‟s own culture is the criterion
against which all other cultures must be judged. It is almost always used in a negative sense to describe
attitudes that refuse to recognize the validity of values that differ from their own. It is difficult to avoid
some measure of ethnocentrism as many cultural values are considered to be universal values or truths.

c) A good-news letter also known as an indirect letter conveys a message of good news
(promotion, transfer to a better place) written by a senior member to the junior one of an organization.
(51)

Such a letter is tuned with the expression of feelings of congratulations. Unlike in a bad-news latter,
the happy aspect of the message is expressed without any delay

d) Members of high-context cultures, particularly Asians, are more collectivist. They emphasize
membership in organizations, groups, and teams; they typically resist independence because it fosters
competition and confrontation instead of consensus. In contrast, members of low-context cultures,
particularly Americans, tend to value individualism. They believe that initiative and self assertion result
in personal achievement. They believe in individual action and personal responsibility, and they desire
a large degree of freedom in their personal lives.

e) Managing Publicity Communication has already been argued that communication is a


two-way process in which the sender and the receiver both play prominent roles. But in Publicity
Communication, the sender apparently plays the significant role to persuade the public at large
(receiver) to understand what has been said. Here the receiver plays a receptive role trying to
understand the contents of the message released by the sender. Like in regular communication,
the public assimilates the useful contents. Such a communication, in which the receiver does not
play a direct role, also generates feedback This is what happens in a PRESS RELEASE
(52)

Section -'B' : Marketing

6. Read the following case carefully and answer the questions that follow:

McDonald having 14,000 branches in USA is a multinational company operating as a fast food chain in the world. It has
more than 32,000 outlets in 121 countries around the globe. It serves about 46 million customers every day. It has recently
offered 50 thousand additional jobs in one day. In its bid to expand its chain across the globe, it entered India in the late
1990s. It set the target to serve 200 million prosperous middle-class people in India and set up 150 outlets there by 2008.

McDonald as the giant fast food chain is popular in the world for hamburgers made mostly of beef. It is the world‟s biggest
user of beef. Beef is prohibited to Hindus, the majority population in India. Likewise, pork is restricted to Muslims, the
second biggest community in India. In addition, there is a sizable population of vegetarians that also include staunch
followers of Jainism.

Well aware of this fact, McDonald offered new product line to Indian market. Maharaja Mac was made of mutton, and
McAloo Tikki Burger was made of potato. McDonald carefully segregated its product into vegetarian and non vegetarian
categories.

When McDonald had just begun positioning itself into the Indian market, it had to face a law suit back in USA. This led to
street protests and pressures from consumers and political parties to close down its operations in India. It hit the image of
McDonald badly.

Questions:
a) What is the marketing mix of McDonald? 5
b) If McDonald is planning to enter Nepalese market, what cultural factors should it consider? 5

Answer No.6

a) As stated in the Case, McDonald‟s is a popular big multinational Company operating in many countries, expanding its
chain to new markets. One of the famous items- hamburgers is being sold in different combination for vegetarians and non
vegetarians and to Hindus, Buddhists, Muslims, Christians etc. Well aware of cultural differences, market segmentation and
product mix is done differently. E.g. Maharaja Mac (of mutton), McAloo Tikki Burger (of vegetables), use of beef-mutton-
pork (differently to different religions).

Generally product is sold at higher standard, expensive retail outlets. Varitiy of products to different segments and central
and local promotions are the other elements of its marketing mix.

b)
The following cultural factors need to be considered for feasibility of McDonald‟s in Nepalese market:
 First of all conduct marketing research +environmental analysis.
 Most cultural issues of Indian market also apply to Nepalese market.
 Emerging eating out habit in urban societies of Nepal.
 Use of red meat consumption is decreasing.
 Brahmins reject buff whereas most of the ethnic groups prefer buff for its low price and taste.
 Changing life styles of urban people.
 Religious neutrality exits in Nepal but majority people are Hindu. Hindus say no to beef whereas Muslim hate pork. On
the other hand Jains are vegetarians.
 Use of public media to convince the people about its product line and product mix.
 Slight modification in marketing mix as well as distribution system for nepalese market.
 Be well aware of street protests, pressure of consumer and political parties.

7.
a) Differentiate between the selling concept and marketing concept. 5
b) How does environment influences industrial development in Nepal? 5

Answer No 7
a) The differences between the selling concept and the marketing concept are as follows
i. The selling concept is a management philosophy based on the premises that consumers generally do not buy products
unless the organization itself puts aggressive selling efforts to create awareness and interest of the consumer in its products
and makes them inclined positively to buy these products. Whereas the marketing concept is a management philosophy that
focuses on the needs and wants of the customers of the target markets and the organization needs to find products and
(53)

services that will prove to be a useful solution to solve the requirement of such needs and wants to deliver the satisfaction
of the customers.

ii. The selling concept helps in getting business. While the marketing concept regards customers as the pivotal point in the
entire marketing plan. The customer‟s needs and wants are supreme. The customer is the boss.

iii. The selling concept focuses on the needs of the sellers, whereas the marketing on concept focuses the needs of the buyer.

iv. The selling concept starts with the existing products and services and tries to fit into the needs of the customers whereas
the marketing concept starts with the proper understanding of target customer‟s needs and wants and then developing
products which would help customers to use and derive satisfaction.

b) Environment changes bring uncertainty in the industrial development. The environmental forces can be political,
economic, social and technological.
(i) Political changes in Nepal have the following impact on Nepalese industrial sector:
 Increasing labor agitation and strikes in industries;
 Close down of industries;
.
(ii) Economic changes have the following impact on Nepalese industrial sector:
 Rise in inflation rate and rise in price.
 Increase in imports and decrease in exports.
 Capital flight due to unsuitable investment climate.

(iii) Social Changes have the following impact on Nepalese industrial sector:
 Increase or decrease in size of population which affect size of market.
 Migration of population.
 Changes in attitudes, values and beliefs.

(iv) Technological changes have the following impact on Nepalese industrial sector:
 New technology replaces old technology
 Technology transfer increases
 New materials and processes become possible.

8.
a) Write a short note on market segmentation and explain its benefits to marketers. 5
b) Explain the significance of marketing information system. 5

Answer No.8
a) Market segmentation is a process of dividing the total market into several smaller groups, such that the members of each
group are similar with respect to the factors that influence demand .By dividing the total market in distinct smaller groups
helps to identify needs and characteristics of buyers. Mass marketing, product variety marketing, individual marketing and
target marketing are some of the common approaches found in dealing with the market.
Benefits of market Segmentation:
To achieve effective market segmentation requires measurability, accessibility, profitability and heterogeneous needs and
characteristics in the market. A proper marketing mix and programs through appropriate segmentation strategy provides the
following benefits:
- Identification of profitable markets
- Market specialization
- Effective use of available resources
- Efficient monitoring of market changes, etc.

b) Successful marketting planning and formulation of strategy requires supporting information and relevant data. No
marketing company can function effectively without appropriate and timely information on each and every aspect of
marketing like the consumer, the market, the competition and the environment. The decision making process becomes easier
with relevant information.
Marketing managers need a continuous flow of information to respond to the environmental changes and to shape and
reshape the marketing mix in order to earn long term business. Marketing department has to organize and distribute a
continuous flow of information to its marketing managers by studying their information needs and designing marketing
information system to meet these needs.
Marketing information system is an organized way of continually gathering and analyzing data to provide relevant
information needed to prepare marketing plan and program. In the words of Philip Kotler, “a marketing information system
(54)

consists of people, equipment and procedures to gather, sort, analyze, evaluate and distribute needed, timely and accurate
information to marketing decision makers.” The significance of marketing information system is:
i. It helps marketing planning by making available and reliable information about the external
environment and the internal realities of the company.
ii. It helps to take advantage of marketing opportunities and to counter marketing threats.
iii. It helps early spotting of changing trends and provides marketing intelligence to the firm.
iv. It facilitates the development of action programs for achieving goals.
v. It helps the firm to adjust its products and services to the needs and tastes of target customers.
vi. It helps the firm to control its marketing activities.

9.
a) Describe briefly the steps of new product development process. 5
b) Describe Distribution structure for consumer goods. 5

Answer No.9
a) New product development is a time consuming and painful process. It steps in this process are:
i. Idea generation: New product development starts with an idea. New product ideas come from a variety of sources i.e.
customers, scientists, competitors, employees, channel members, investors, marketing consultants, universities, business
journals, seminars etc. Various techniques have been developed and used for idea generation i.e. attributes listing, forced
relationship, morphological analysis, need/problem identification, and brainstorming.

ii. Idea screening: The purpose of screening is to drop poor ideas as early as possible. The total ideas are categorized into
three groups i.e. promosing ideas, marginal ideas and reject ideas.

iii. Concept development and testing: It is the method of trying to guage buyer interest before actual product has been
developed. A product idea can be turned into several concepts. In fact, consumers buy product concepts. Concept testing
entails presenting consumers with an elaborated version of the concept. Intercept interview technique is mainly used for
concept testing. Generally, this type of interview focuses on buying intent, frequency of purchase of the product, uniqueness
of the concept, the price, and the problem solving attributes of the product.

iv. Marketing Strategy development: After testing, the product manager must develop a marketing strategy for the new
product. Such a marketing strategy plan includes the target market size and structure, possible positioning for the product,
preliminary estimate of sales, market share, the product‟s price, distribution, promotion and marketing budget, long run
sales and profit goals and marketing mix program overtime.

v. Business analysis: This step is essential in the total process of new product development because several vital decisions
regarding the product are expanded into business proposal. These decisions should be financial as well as marketing point of
view. Management should analyze various activities in this step i.e. identify product features, estimate market demand,
competition and product‟s profitability, establish a program to develop the market and assign responsibility for study of the
feasibility.

vi. Product development and testing: In this step, the idea-on-paper is converted into a physical product. Having completed
the business analysis, it moves to R&D or engineering to be developed into a physical product. Product testing may be
conducted through an expert panel, central location testing, in-home test etc.

vii. Market testing: After management is satisfied with functional performance, the product is given a brand name,
packaging and put to market test. The new product is introduced in selected market to learn how large the market is and how
consumers and dealers react to handling, using and repurchasing the product.
viii. Commercialization: Full-scale production and marketing programs are planned and the product is launched in the target
market.

b) Distribution structure for consumer goods should be as follows:


Four channel structures are widely used in the marketing of consumer goods. The commonly used channels for consumer
goods are as follows:
(55)

Channel Structure for Consumer Goods.

Manufacturer of Consumers Goods

1 2 3 4

Agent

Wholesaler Wholesaler

Retailer Retailer Retailer

Consumer

1. Manufacturer - Consumer Channel


This is a direct channel which involves no middleman. Therefore, this channel is also known as Zero-level channel of
distribution, Here, the manufacturer sells goods and services directly to the consumers through indoor sales process or
outdoor sales process.

2. Manufacturer-Retailer-Consumer Channel
In this channel structure, one-level of middlemen is used i.e., retailers, Manufacturer sells to the retailers and retailers
sell to the ultimate consumers. Here, retailers function as middleman.

3. Manufacturer-Wholesaler-Retailer-Consumer Channel
In this channel structure, two-levels of middlemen are used. They are wholesalers and retailers. In this system,
manufacturer sells goods to the wholesalers, wholesalers sell to the retailers, and retailers sell to the ultimate consumers.

4. Manufacturer-Agent-Wholesaler-Retailer-Consumers Channel
In this channel structure, three levels of channel middlemen are used to reach the ultimate consumers. They are agent,
wholesaler and retailer. Here, agent collects goods from the manufacturer and distributes to the wholesaler, wholesaler
sells them to the retailers, and the retailers sell to the consumers.
The second, third, and fourth channel of distribution are indirect channels of distribution since independent
middlemen are used to reach the final consumers. In direct channel system, the manufacturer exercises maximum level
of distribution control. While in indirect channel system, the manufacturer looses controlling power over distribution
functions.

10. Briefly explain the following: (5×2=10)

a) Telemarketing.
b) Elements of marketing communication.
c) Test marketing.
d) Resolution of channel conflicts.
e) Promotion mix.

Answer No.10
a) Telemarketing
It is marketing done by using telephone and call centers to attract customers. It sells to existing customers and provides
service by taking order and answering questions. It is becoming popular as business houses carryout telesales, telecoverage,
teleprospecting, customer services and technical support. Customers get improved satisfaction from telemarketing as a direct
marketing tool. Telemarketer has reduced the need of personal selling significantly.

b) Elements of marketing communication


The elements of marketing communication are as follows:
Source: The communication process begins with a source who is generally the sender of the message. The source is
generally the manufacturer, marketing channel members who initiate and intend to send a communication.
Encoding: It is the process of designing message by putting thoughts and ideas in symbolic language.
Message Channel: It is generally called media. This is the path through which the message moves from one party to another.
Media may be audio, video or print.
(56)

Decoding: Decoding is the process by which the receiver assigns meaning to symbol transmitted by the sender.
Receiver: The receiver is a person, a group or organization to which the communication is directed.
Feedback: It provides information to redesign communication to meet changing needs of marketing.

c)Test Marketing:
It is related with testing commercial viability of new product in the context of a market segment on a limited scale.
Marketing is done to have a reasonable estimate of sales and market share to make necessary changes in the marketing mix.
The product is sold in selected areas with the help of local level promotion in few shops. Test marketing is periodically
evaluated in terms of trial rate, advertising effectiveness, and repeat purchase rate.

d) Resolution of channel conflicts


Channel members usually specialize in particular marketing functions. Such as, manufacturers focus on production and
national promotion, wholesalers perform distribution task, and retailers specialize in distribution and promotion at the
consumer level. Such specialization creates conflicts among channel members. As a result, conflict resolution among
channel members becomes essential.
In practice, there are four methods of resolving channel conflicts:
 Resolution through Mitigation
 Resolution through Conciliation
 Resolution through Arbitration
 Resolution through Regulation.

 Conflict Resolution through Mitigation


Mitigation is a method of resolving disputes through mutual understanding between the parties without using any mediator.
It is possible only when both the parties honestly realize that the good faith between them must be restored for their long-
term relationship. In this case, both the parties actively and mutually take actions to restore their relationship without asking
the help of any third party. This method of resolving conflict is regarded as the best method.

 Conflict Resolution through Conciliation


Conciliation agreement between parties to resolve disputes by asking a third party to mediate differences voluntarily. The
third party does hot have any power to force the conflicting parties. If the agreement is reached, a conciliation statement
based on the signed agreement is recorded.

 Conflict Resolution through Arbitration


Under this method, the conflicting parties gather together with an arbitrator. They disclose all their opinions and then make
an agreement to continue the business with full understanding and confidence. This method permits them to continue their
business without damaging the goodwill of either of the parties. Timely resolution of the channel conflict is another positive
point of this method of resolution.

 Conflict Resolution through Regulation


Under this method, conflicts are resolved through law. All the conflicting parties take the help of law and go to the court for
the resolution of their disputes. However, this method of conflict resolution is an expensive, time consuming, and harmful to
all conflicting parties; It may damage the goodwill of both the parties. This method of conflict resolution is not popular and
practical. This method is adopted only when the channel conflict between the parties is difficult to settle through arbitration.

e) Promotional mix:
Promotion is the fourth element in the marketing mix, which is generally divided into five different tools. They are:
 Personal selling: It consists of executing sales through salespersons.
 Advertising: Any paid form of non-personal presentation to promote products and services by
identified sponsor.
 Sales promotion: Short term incentives that encourage people to buy a product/service.
 Publicity: An unpaid form of non-personal presentation of news and information about products or
services to prospective customers.
 Direct marketing: Non-personal presentation to the prospective customers to which they can
respond directly and quickly.
The Institute of Chartered Accountants of Nepal
Suggested Answers of Income Tax and VAT
CAP II Examination – June 2011

8. Answer the following with reference to the Indian Income Tax Act, 1961.
a) What are the bases for the dividend income to be includible in the total income of the assessee?
2
b) Mr. Rama Swami is an Indian citizen. He left India for the first time on May 10, 2004. During the previous year
2008/09, he came to India once on June for a period of 10 days. Determine his residential status for the assessment year
2009/10 on the assumption that financial year is previous year. Does it make any difference if Mr. Rama Swami is not
an Indian citizen?. 3
c) Income earned during a previous year is charged to tax in the financial year next following it.” Write in brief if there are
any exceptions to this normal rule.
5

Answer No.1
a) Dividend income is includible in the total income of the assessee on the following basis:
Nature of dividend Year of chargeability
1. Final dividend The previous year in which it is declared in the AGM.
2. Interim dividend The previous year in which it is unconditionally made available.
3. Deemed dividend The previous year in which it is paid or distributed.

b) Since Mr. Rama Swami comes to India only for 10 days in the previous year 2008/09, he does not satisfy any of the
basic conditions mentioned in sec 6(1). Therefore, he is non-resident in India for the assessment year 2009/10.
Even if he is not a citizen of India, he will remain a non-resident for the assessment year 2009/10.

c) The exceptions to the said rule are as follows:


i. In the case of shipping business of a non-resident, the ship is allowed to leave the port only if the tax due thereon has been
paid or arrangement has been made for payment thereof.(Sec.172)

ii. If a person is leaving India, the total income of such individual for the period from the expiry of the previous year for that
assessment year up to the date of his probable departure is chargeable to tax in that assessment year. (Sec.174)
iii. In the case of a person likely to transfer property to avoid tax, the total income of such person for the period from the
expiry of the previous year for that assessment year to the date when the Assessing Officer starts preceding is chargeable to
tax in that assessment year. (Sec.175)
iv. Where any business is discontinued in any year, any sum received after the discontinuance is deemed to be the income of
the recipient and charged to tax accordingly in the year of receipt. (Sec.176).

9.
a) Mr. Yubaraj Thapa, a bank officer, retired from his job in the year 2065/66. He received Rs. 4,50,000 as retirement
payment. The payment was made through an approved retirement fund. From the Shrawan 1, 2066 he is entitled to get a
pension of Rs. 21,000 per month. He also entitled to get one month's pension as dashain bonus. Mr. Yubaraj received
interest of Rs. 50,000 from a fixed deposit from Rastriya Banijya Bank. He has also received a dividend of Rs. 95,000 from
Nabil Bank. He started a consultancy business in the month of Shrawan 2066 and earned net profit of Rs. 25,000 in the year
2066/67.
His wife Mrs. Lolita is a school teacher in a government school and getting Rs. 10,000 monthly salary. She is also operating
a boutique since last five years and the net profit and the annual sales for the year 2066/67 are Rs. 1,50,000 and Rs.
12,45,000 respectively. Mrs. Lolita owns a house located at Putalisadak and the rental income of the house is Rs. 50,000 per
month. The house was rented to a commercial bank. She also does the assignment of question paper setting and evaluation
of answers and she received Rs. 1,05,000 during the year 2066/67.
Mr. Yubaraj and his wife have not selected the option as a couple for tax returns purpose. Calculate the tax liability of Mr.
Yubaraj and Mrs. Lolita and also state their responsibility to file the income tax return for the year 2065/66 and 2066/67. 12

b) Smart Pvt. Ltd. purchased a piece of land on Ashwin 25, 2064 for Rs. 10,00,000 and incurred an expenditure of Rs.
1,50,000 on registration and brokerage. It constructed a building on the land costing Rs. 25,00,000. The building was ready
for use on Shrawan 25, 2065. The depreciated value in Block A was Rs. 10,00,000 at the end of year 2064/65 excluding the
cost of the building under construction. During the year 2065/66 the company capitalized Rs. 60,000 as repair and
improvement cost in Block A.
On Shrawan 1, 2066, the company sold the land and newly constructed building for Rs. 40,00,000. The market value of land
on that date was Rs. 15,00,000 and that of the building was Rs. 25,00,000. You are required to calculate taxable gain for the
disposal of land and building for the year 2066/67.
5

NMG P.T.O.
(58)

c) In the occasion of Deepawali 2067, Himal Group organized a nationwide song competition and Mr. Ghanashyam
wins a first prize of Rs. 5,00,000. The organizer of the program wants to deduct TDS @ 25% on the prize amount and pay
the balance to him.

d) Mr. Ghanashyam was of the view that prize is related to the work of art and TDS is not applicable on this payment.
As a tax consultant, you are required to advice to Mr. Ghanashyam whether TDS is applicable in this payment or not also
mention the relevant provisions of Income Tax Act 2058.
3

Answer No.2
a) Calculation of Tax Liability for the year 2065/66
Mr. Yubaraj was the bank officer in the year 2065/66 his employer must have deducted TDS from his salary and
deposited to the revenue. The payment from approved retirement fund Rs. 4,50,000 is exempt (as it is less than
5,00,000) from tax as per the provisions of section 65.

There is no information regarding the total income of Mrs. Lolita for the year 2065/66, therefore we could not calculate
her tax liability for this year.

Calculation of Tax Liability for the year 2066/67

Calculation of Tax Liability of Mr. Yubaraj


Pension income (21000*12) 2,52,000
Dashain Bonus 21,000
Net Profit from consultancy business 25,000
Total assessable income 2,98,000

Tax Liability:
Upto Rs. 1,60,000 @1% 1,600
Additional exemption for pension 25% of Rs. 160000
or actual pension whichever lower = Rs. 40,000 Nil
Balance Rs. 98,000 @ 15% 14,700
Total Tax Liability 16,300

Calculation of Tax Liability of Mrs. Lolita


Salary Income (10,000*12) 1,20,000
Dashain Bonus 10,000
Income from boutique 1,50,000
Income from question paper setting and evaluation 1,05,000
Total assessable income 3,85,000

Tax Liability:
Upto Rs. 1,60,000 @1% 1,600
First Rs. 1,00,000 @ 15% 15,000
Balance Rs. 1,25,000 @ 25% 31,250
Total Tax Liability 47,850

Responsibility to file income tax return:

Mr. Yubaraj is required to file income tax return as follows:


i. He is not required to file income tax return for the year 2065/66 as he has only salary income and his employer must have
deducted TDS from his salary.
ii. He should file income tax return for the year 2066/67 as he earned income from pension and consultancy business.

Mrs. Lolita is required to file income tax return as follows:


i. She is required to file income tax return for the year 2065/66 as she has boutique business in that year.

ii. She is also required file income tax return in the year 2066/67 as she has salary income as well as business income.

Notes:
i. Bank interest and dividend are final withholding income of Mr. Yubaraj therefore these incomes are not included in his
income.
ii. House rent income of Mrs. Lolita is also a final withholding income.
iii. It is assumed that Mrs. Lolita will get dashain bonus equal to one month‟s salary.
(59)

iv. TDS shall not be deducted while making payment for the setting of question papers and evaluation of answers but this
income shall be included while calculating the taxable income of the recipient.

b. Calculation of taxable gain or loss from the disposal of land:

Selling price of land 15,00,000


Cost of land:
Purchase price 10,00,000
Registration and brokerage 1,50,000
Total cost of land 11,50,000
Gain on disposal of land 3,50,000

Calculation of taxable gain or loss from the disposal of building:

Depreciated balance at the end of the year 2064/65 10,00,000


Absorbed addition for the year 2065/66 (cost of building) 25,00,000
Depreciable basis at the end of 2065/66 35,00,000
Less: Depreciation for the year 2065/66 @5% 1,75,000
Depreciated balance 33,25,000
Add: Repairs capitalized 60,000
Depreciated balance at the end of 2065/66 33,85,000
Less: Selling price of building 25,00,000
Depreciable basis of building 8,85,000

Note 1:
Taxable gain from the disposal of land is Rs. 3,50,000 and there is no taxable gain from the disposal of building. The sales
value of building shall be deducted from the total amount of block and the remaining amount Rs. 8,85,000 will be the
depreciable balance of Block A.

c. As per section 88ka, TDS @ 25% shall be deducted in any windfall gain. Therefore, the organizer has to deduct TDS @
25% while making this payment to Mr. Ghanashyam.
The contention of Mr. Ghanashyam is not correct in this case as this amount has not exempted by Nepal Government from
windfall gain tax.

10.
a. As a tax consultant, you have been enquired of the implication on income tax on the following transactions: (2+1+1+1=5)
i) Kanyam Tea Estate Ltd. is a co-operative society established under the Co-operative Society Act 2048. In that society,
40 farmers of Kanyam region are involved in tea gardening and processing. In the financial year 2067/68, Kanyam Tea
Estate has earned Rs. 1 Crore. State with reason on the taxability of such income earned. Further, what will be the impact on
income tax if dividends are declared by that society?
ii) Income of the Securities Board of Nepal.
iii) Foreign investors have earned Rs. 50,00,000 as foreign technology and management service fee from the industry
established in the special economic zone
iv) Income earned by the Entities listed in the securities market and engaged in the business of production, Tourism sector.

b. As a tax consultant, you have been enquired by the clients on the depreciation facility for the purpose of income tax on
following cases: (2×2.5=5)
i) Assets, required for power generation for its industry, capitalized by the production oriented industries.
ii) If a person who wants to issue the tax invoices using fiscal printer and cash machine.

c. What do you understand by the expenses of Domestic or Personal nature? 5


d. Define Income, Windfall gain, Assessable Income and Taxable Income. 5

Answer No.3
a)
i. As per section 11(2) of the Act, Income of a co-operative society, having incorporated under the Cooperative Society Act,
2048 earned from any or more activities specified including tea gardening and processing is exempted from income tax.
Hence Rs. 1 crore is exempted from income tax.
Further as per the same section, dividend distributed by such cooperatives is also exempted from tax and hence no need to
deduct TDS while making payment to its members.
(60)

ii. As per section 10(Chha) of the Act, income of Securities Board of Nepal earned as per its objectives is exempted from
income tax.
iii. As per section 11(3Ka)(gha) of the Act, in case of income earned by a foreign investors towards foreign technology and
management service fee and royalty income from the industries established in Special Economic Zones, 50% exemption in
the applicable tax is available.
iv. As per section 11(3Chha) of the Act, entities engaged in the business of Production, Tourism, Hydroelectricity
generation, distribution and transmission etc. and listed in the securities market get 10% exemption in applicable income
tax.
b)
i. As per Sec 3(3) of schedule 2 of Income Tax Act 2058, Production oriented industries if capitalize the assets required for
power generation for its industries can claim 50% of total capitalized amount as depreciation in the year of capitalization.

ii. As per Sec 3(4) of schedule 2 of Income Tax Act 2058, if a person wants to issue the tax invoice by using fiscal printer
and cash machine, then total amount spent for such fiscal printer and cash machine can be claimed as depreciation in the
year of purchase.

c) Expenses of domestic nature or personal nature are not allowed for deduction for the purpose of income tax. The
clarification clause of section 21 explains the nature of domestic or personal nature as follows:
- Interest incurred on amount borrowed to the extent to which it is used for personal purpose.

- Expenses of very personal nature incurred for an individual in providing residence, meals, refreshment, entertainment or
other leisure activities.
- Expenses incurred by an individual on conveyance from residence to office and office to residence.

- Expenses incurred on an individual for clothing which is also suitable to wear outside the work

- Expenses incurred on education and training. But the expenses incurred on such training directly relating to the
business, yet not leading to a degree or diploma, are allowed for deduction.

- Any expenses incurred to make a payment to a natural person or the expenses incurred for a third person, except in and
to the extent of the following conditions:
 The payment is included in calculating the income of the individual- such as house rent, driver facility, gardener,
servant, telephone in residence provided to an employee. If the expenses are included in the taxable income of the
individual, the expenses are allowed for deduction to the person.
 The individual makes a return payment of an equal market value to the person as a consideration for the payment.

 Small amount incurred in this respect for which keeping an individual account is impracticable, for tea, stationery,
awards, emergency medical facility or any other expenses as provided by IRD up to Rs. 500 at a time.

d) As per section 2(ja) of the Act, income is the total amount received from business, employment, investment or windfall
gain. Income denotes the total receipts, which are included in calculation of taxable income (sec. 7, 8 and 9).

As per section 2(ja1) of the act, windfall gains includes income from lottery, gift, prize, tip, win-rings and other similar
casual incomes. As per section 6 of the act, assessable income is the sum of income from business, employment, investment.
In case of a resident person, it includes all income from any country for the concerned income year and in the case of non
resident person, it includes the income having source in Nepal. As per proviso to section 6 of the act, assessable income
does not include any concession under section 11 and income of an Approved Retirement Fund under section 64.
As per section 5 of the act, taxable income is determined by deducting the amount of donation, subject to limitation under
section 12 and 12ka and contribution to approved retirement fund subject to limitation under section 63 from the assessable
income.

4.
a. XYZ Co. Ltd purchased an Accounting Software for Rs. 50,00,000 with the life span of 10 yrs and 5 months. It was
installed on 1st Shrawan 2065. Also, the Company purchased Inventory Management Software for Rs. 31,50,000 with the
working life of 10 yrs and 6 months. This was put in use in the month of Baisakh 2066.
Compute the allowable depreciation for Tax purpose for Financial Year 2065/066. 5

b. Details of annual estimated tax and withholding tax of A & Co. are as follows:
Estimated Tax
Poush end Rs. 10,00,000
Chaitra end Rs. 12,00,000 (Re-estimated)
(61)

Ashad end Rs. 12,00,000 (Re-estimated)

Withholding Tax details:


Upto Poush end Rs. 20,000
Magh to Chaitra end Rs. 10,000
Chaitra to Ashad end Rs. 15,000

Compute the total Advance Tax to be paid for each installment. 5

Answer No.4
a) These intangible assets fall under Block “E”. As per section 3 of schedule 2 of Income Tax Act, the depreciation of
intangible assets shall be charged as follows:
Cost of the software/ useful life= Amount of depreciation per year (straight line Basis).
In case the year of useful life is in fraction of a year, the nearest half year shall be considered.

According to section 1(2)(kha) of schedule 2, each intangible assets should be disclosed separately.

Accounting software:
Cost Rs. 5,000,000 and the useful life is 10 years 5 months = 10.5 years.
As the software was installed on 1st Shrawan 2065, for the financial year 2065-66, depreciation shall be available for the
whole year.
Thus, depreciation for 2065-66 shall be:
Rs. 5,000,000 /10.5 = Rs. 4,76,190.48

Inventory Management software:


Cost Rs. 3,150,000 and the useful life is 10 years 6 months = 10.50 years.
As the software was installed on Baisakh 2066, as per section 5 of the schedule, depreciation shall be available on 1/3 rd of
the amount for financial year 2065-66.

Thus, the amount of depreciation shall be:


Rs. 3,150,000/10.5/3 = Rs. 100,000.

Thus, the total depreciation for 2065-66 shall be: Rs. 4,76,190.48 + Rs. 100,000= Rs. 5,76,190.48.

b) 1st Installment (Poush end)


= 40% of estimated tax – withholding tax – Advance tax
= 40% of Rs. 10,00,000 – Rs. 20,000 – 0
= Rs. 3,80,000

2nd Installment (Chaitra end)


= 70% of Rs. 12,00,000 – Rs. 30,000 – Rs. 3,80,000
= Rs. 4,30,000

3rd Installment (Ashad end)


= 100% of Rs. 12,00,000 – Rs. 45,000 – Rs. 8,10,000
= Rs. 3,45,000

Total Advance Tax paid:


Withholding Tax Rs. 45,000
1st Installment Rs. 3,80,000
2nd Installment Rs. 4,30,000
3rd Installment Rs. 3,45,000
Total Rs. 12,00,000
5.
a. Sharma & Sharma Co. has been a self employed VAT registered trader since 2064 B.S, and is in the process of
completing the VAT return for the month ended 31 Baisakh 2068. The following information is relevant to the completion
of the VAT return;
i) Sales invoices totaling Rs. 44,00,000 (excluding VAT) were issued to VAT registered customers in respect of VAT
sales. The firm offers a 5% discount for prompt payment.
ii) Sales invoices totaling Rs. 16,92,000 were issued to customers that were not registered for VAT. Of this figure, Rs.
51,700 was in respect of zero-rated VAT sales.
iii) During the month of Baisakh 068, goods amounted to Rs. 11,20,000 were purchased. Of this figure, Rs. 80,000 was
used for Mr. Sharma‟s private purpose.
(62)

iv) On 15 Baisakh 068, a Toyota Hilux vehicle was purchased for Rs. 24,00,000. The cost is inclusive of VAT.
v) During the month ended 31 Baisakh 068, Rs. 40,000 was spent on mobile telephone calls, of which 30% relates to
private calls.
vi) On 17 Baisakh 068, an office equipment was purchased for Rs. 6,00,000. The purchase was partly financed by a bank
loan of Rs. 5,00,000.
Unless stated otherwise, all of the figures are exclusive of VAT.
Calculate the amount of VAT payable by Sharma & Sharma Co. for the month ended 31 Baisakh 2068.
10

b. XYZ Pvt. Ltd. imports/purchases the taxable raw materials such as Iron Ingots and Steel Plates to manufacture them into
different furniture items as well as agriculture tools.
Details of total imports excluding Input VAT are as follows:
Cost of Iron Ingots declared by XYZ Pvt. Ltd. Rs. 3,00,000
Revised cost fixed by Customs Officer Rs. 3,50,000
Freight Rs. 50,000
Insurance Rs. 5,000
Import Duty @ 6%
In addition, it has also purchased following raw materials:
Steel plates (exclusive of VAT) Rs. 1,00,000
Woods (exclusive of VAT) Rs. 35,000

The products manufactured out of the above raw materials are sold out with the details as given below:
Agriculture tools (VAT exempt items) Rs. 2,50,000
Furniture Rs. 3,00,000
Find the ratio between taxable and non taxable sales and calculate the amount of Input VAT that XYZ Pvt. Ltd. is entitled to
claim under sec 17 (3) of VAT Act. 10

Answer No 5
a) Statement of VAT Return for the month ended 31 Baisakh 2068:
Sales (Rs. 44,00,000 * 13%) Rs. 5,72,000
Sales (Rs. 16,92,000 – Rs. 51,700 = Rs. 16,40,300* 13%) Rs. 2,13,239
Sales (Rs. 51,700 * 0%) Rs. 0______
Total output VAT (A) Rs. 7,85,239

Goods Purchased (Rs. 11,20,000 - Rs. 80,000) * 13% Rs. 1,35,200


Vehicle (Rs. 24,00,000 * 40% * 13/113) Rs. 1,10,442
Mobile expenses (Rs. 40,000 * 70%) * 13% Rs. 3,640
Office Equipment Purchase (Rs. 6,00,000 * 13%) Rs. 78,000_
Total Input VAT (B) Rs. 3,27,282

Net VAT payable (A-B) Rs. 4,57,957


b.
Computation of Input VAT amount on import:
Cost of Iron Ingots Rs. 3,50,000
Freight Rs. 50,000
Insurance Rs. 5,000
Total Rs. 4,05,000
Import Duty Rs. 24,300
Total cost for VAT purpose Rs. 4,29,300

Input VAT on import = 13% of Rs. 4,29,300


= Rs. 55,809
Computation of Input VAT amount on local purchase:
Steel plates Rs. 1,00,000
Woods Rs. 35,000
Total cost Rs. 1,35,000

Input VAT = 13% of Rs. 1,35,000


= Rs. 17,550
Total input VAT:
Input VAT on Import = Rs. 55,809
Input VAT on local purchase = Rs. 17,550
Total = Rs. 73,359
(63)

Computation of sales:
Taxable sales (Furniture) Rs. 3,00,000
VAT Exempted sales (Agriculture tools) Rs. 2,50,000
Total sales Rs. 5,50,000

Ratio between taxable and non-taxable sales is 54.546:45.454 say 55:45.


Therefore, XYZ Pvt. Ltd. is entitled to claim only for 55% or Rs. 40,347.45 out of total Input VAT of Rs. 73,359.

Note: Total amount of VAT paid on raw materials could be setoff against the sales of finished products if the finished
product is taxable item. In the question, it is not given that the raw materials purchased are used only for taxable item or
non-taxable item. Thus, the VAT paid on raw materials is proportionately on the basis of sales of taxable and non-taxable
sales.

6.
a. How is the taxable value for second hand or used goods computed? Similarly how the taxable value for wood of national
forest, private and community forest is computed? 5
b. Describe the procedure of Administrative Review in Value Added Tax. 5
c. What are the penalties mentioned in section 29 of the act on the following infringements: (5×1=5)
i) Registration as mentioned in section 10(1) and (2) related infringement.
ii) An unregistered person issuing an invoice or documents showing collection of tax.
iii) On obstruction in inspection by a tax officer.
iv) On infringement of the VAT act and the rules.
v) To erase and edit the data in software of approved computer Billing System.

d. M/s XYZ Drinkers Ltd. has sold a kind of drink to non VAT registered party for Rs.10,000. As per the retail price
published by the same company under the direction of IRD, the retail price is Rs. 12,000. The company has collected VAT
from the party on the amount of Rs.10,000 as it says it has given trade discount to the party and the trade discount can be
deducted to arrive at the transaction value. But the assessing officer insists to collect VAT on the published price. As an
expert, give your opinion on this. 5

Answer No.6
a) As per Rule 33 of VAT Rules, taxable value for dealers in secondhand or used materials is the difference of Sales
amount and Purchase Amount (including VAT). In case the purchase price of every item of used goods exceeds Rs.10,000,
separate records of buying or selling shall be maintained. As per rule 19, in case of registered person dealing in second hand
or used goods is not required to issue tax invoice if the selling price is less than the purchase price and the cost of the
material is less than Rs.10,000. It means if used goods sold in loss (purchase price + additional cost+ VAT paid on purchase
– selling amount without VAT), then the taxable amount is nil and tax invoice need not to be issued. In case a registered
person is observed by Tax Officer that it has not maintained the prescribed records satisfactorily, Tax Officer may impose
VAT on the total selling price of the goods sold by such tax payer, and the tax officer may issue a written order requiring
him to pay such tax along with the next tax return.

As per section 12ka, in case the wood from a national forest is being sold, tax shall be levied on the amount on which
royalty is being calculated or the amount of the auction, whichever is higher. The amount considered for such calculation
shall be on the basis of the earlier of these happenings: auction of wood of the national forest, issue of delivery order or
issue of an order to cut the wood. Similarly, wood of a private area, private forest and community forest, in case it is sold for
business, although royalty is not charged on such sales, for tax purpose, it is determined on the basis given for the wood of
national forest.

b) As per section 31(ka) of the Act, in case a taxpayer is not satisfied with any decision on tax assessment made by a tax
officer, it can file an application to IRD for an administrative review, within thirty days of the receipt of the notice of the
decision.
In case the taxpayer is not able to submit an application within the specified days, it can apply to IRD for extension of the
time, specifying the valid reasons for the delay, within seven days from the expiry of the time. IRD may extend the time for
a period not exceeding 30 days from the expiry of the time.
In case IRD, on scrutiny of the application and the documents submitted by the taxpayer, thinks that the application is
allowable, after noting the fact in a file, it may order a reassessment by the same tax officer or by any other tax officer.

IRD, up to possible, has to decide on the application within 60 days of the date of application received.

In case IRD fails to decide on the application within sixty days, the taxpayer may appeal to the Revenue Tribunal supposing
that IRD has given its verdict against the taxpayer. [0.5]
A person who applies for departmental review has to deposit 100% of the undisputed amount and 1/3 of the disputed
amount before the application is submitted.
(64)

c)
i. Non registration within the time period mentioned attracts penalty Rs.10,000 for each tax period.

ii. If an unregistered person issues an invoices or documents showing collection of tax, then 100% of the tax collected shall
be the applicable penalty.
iii. If obstruction is caused in inspection by a tax officer, then Rs.5,000 for each time shall be applicable as penalty.

iv. On infringement of the VAT Act and the Rules, then Rs.1000 for each time shall be the applicable penalty.

v. If the tax payer has erased and edit the data in software of approved billing system, then Rs. 5 Lac will be the applicable
penalty.
d) As general rules, the taxable value of an item is net of discount allowed in invoice. But, as per section 14(7) of VAT
Act, in case a registered person sales a notified item to a non-registered person, then the taxable value shall be the retail
published price. If discount is to be given that should be given after charging VAT.
Here notified items means the items for which IRD has notified that a producer of the items is required to publish its retail
price. Hence the contention of the assessing officer is correct.

You might also like